Exam 2 Practice Questions

¡Supera tus tareas y exámenes ahora con Quizwiz!

During pregnancy a woman should add _____ calories to her diet.

300

Which of the following are considered presumptive indications of pregnancy? (Select all that apply). A.) Amenorrhea B.) Nausea and vomiting C.) Abdominal enlargement D.) Ballottement E.) Positive pregnancy test F.) Auscultation of fetal heart sounds

A.) Amenorrhea B.) Nausea and vomiting

A client who has experienced a spontaneous abortion at 8 weeks asks the nurse why this happened. What would the nurse include in a response to address the most common cause of "miscarriage?" A.) Chromosome abnormalities B.) Environmental teratogens C.) Excessive activity D.) Substance abuse

A.) Chromosome abnormalities

When counseling a teenage woman who is 8 months pregnant and seeking health care for the first time, the nurse is aware that there are alternatives available to this woman. What would these be? (Select all that apply). A.) Terminate the pregnancy. B.) Continue the pregnancy and place the infant for adoption. C.) Keep the infant.

B.) Continue the pregnancy and place the infant for adoption. C.) Keep the infant.

Which nursing diagnosis has the highest priority for a client with a missed abortion who has developed disseminated intravascular coagulopathy (DIC)? A.) Anticipatory Grieving B.) High Risk for Deficient Fluid Volume C.) High Risk for Injury D.) Spiritual Distress

B.) High Risk for Deficient Fluid Volume

The fetal heartbeat should be visible on ultrasound by the: A.) Fourth week following the last menstrual period. B.) Sixth week following the last menstrual period. C.) Ninth week following the last menstrual period. D.) Twelfth week following the last menstrual period.

B.) Sixth week following the last menstrual period.

If the first day of a woman's last normal menstrual period was May 28, what is the EDB using the Nagele rule?

Count back 3 months and add 7 days: March 7 (always give February 28 days).

After receiving magnesium sulfate, a client develops signs of toxicity. What should the nurse be prepared to administer? A.) Oxygen B.) Epinephrine C.) Potassium chloride D.) Calcium gluconate

D.) Calcium gluconate

The client has come to the clinic for her first prenatal visit. During the pelvic examination, the examiner indicates that the vaginal mucosa has a bluish color. The nurse documents which assessment as positive? A.) Hegar's sign B.) Goodell's sign C.) McDonald's sign D.) Chadwick's sign

D.) Chadwick's sign

At 20 weeks; gestation, the fundal height would be _______.

at the umbilicus.

At the first prenatal visit, the client reveals that her last menstrual period began March 18 (03.08). The nurse calculates her estimated date of delivery to be ________.

12.25

A client comes to the clinic for her first prenatal visit and reports that July 10 was the first day of her last menstrual period. Using Naegele's rule, the nurse calculates the estimated date of birth for the client to be________.

4.17

Magnesium sulfate is used to treat preclampsia. A. What is the purpose of administering magnesium sulfate? B. What is the main action of magnesium sulfate? C. What is the antidote for magnesium sulfate? D. List the three main assessment findings indicating toxic effects of magnesium sulfate.

A. To prevent seizures by decreasing CNS irritability. B. CNS depression (seizure prevention) C. Calcium gluconate D. Reduced urinary output, reduced respiratory rate, and decreased reflexes

Which of the following statements by the pregnant client indicates to the nurse an understanding of the client's nutritional needs during the second and third trimester? Select all that apply. A.) "I will need to increase my intake of protein." B.) "I will need to increase my daily intake by 500-600 calories per day." C.) "I will need to increase my intake of calcium so that it is double my phosphorus intake." D.) "I will need to decrease my intake of iodine." E.) "I will need to increase my iron and may even need a prenatal iron supplement."

A.) "I will need to increase my intake of protein." E.) "I will need to increase my iron and may even need a prenatal iron supplement."

The pregnant client is 7 cm dilated, 100% effaced, and at a +1 station. The fetus is in a face presentation. The nurse concludes that client teaching has been effective when the client's husband makes which statement? A.) "Our baby will come out face-first." B.) "Our baby will come out facing one hip." C.) "Our baby will come out buttocks-first." D.) "Our baby will come out with the back of the head first."

A.) "Our baby will come out face-first." Rationale: Presentation refers to the part of the fetus that is coming through the cervix and birth canal first. Thus a face presentation occurs when the face is coming through first.

Which statement indicates to the nurse that a couple is coping with the stress of infertility treatment? A.) "We are trying to maintain a little romance in our relationship." B.) "My wife was so upset she threw a syringe at me yesterday." C.) "My husband couldn't have an erection when he was supposed to." D.) "We have two or glasses of wine each night to help us relax."

A.) "We are trying to maintain a little romance in our relationship." Rationale: Maintaining a healthy relationship, such as romance, is important during infertility treatments, which can be very stressful. Emotional outbursts, decreased libido, and regular use of alcohol to relax warrant further investigation as possible signs of excess stress.

During a prenatal assessment the client, 3 months pregnant, informs the nurse that she was diagnosed with phenylketonuria as an infant. During the diet assessment the client writes down that she drinks about three or four diet colas a day. One important question the nurse should ask next is: A.) "What type of sweetener is in the colas?" B.) "What time of day do you drink the colas?" C.) "Do you still drink at least two or three glasses of milk a day?" D.) "Do the colas take away your appetite for other foods?"

A.) "What type of sweetener is in the colas?" Drinks sweetened with aspartame should not be used by pregnant women who have phenylketonuria. This can result in brain damage because they lack the enzyme to metabolize it.

A client who admits to crack cocaine use during her pregnancy asks the nurse not to inform the baby's father about the substance abuse. Which response by the nurse is appropriate? Select all that apply. A.) "You must be worried about how he will react to that information." B.) "This is your pregnancy and your body, so I'll keep your information private." C.) "Your baby will probably not survive, so there is no need for him to know." D.) "Have you considered that he deserves to know that the baby may be at risk?" E.) "What reaction do you think the baby's father will have?"

A.) "You must be worried about how he will react to that information." E.) "What reaction do you think the baby's father will have?"

During antepartal teaching, the nurse informs clients that during the second and third trimesters, they should gain weight at a rate of: A.) 1 pound (0.44 kg) per week. B.) 1 pound (0.44 kg) per month. C.) 2 pounds (0.88 kg) per week. D.) 2 pounds (0.88 kg) per month.

A.) 1 pound (0.44 kg) per week. The pattern of weight gain during pregnancy should be about 3.5 pounds (1.6 kg) during the first trimester and then about 1 pound (0.44 kg) per week for the rest of the pregnancy.

The nurse will be concerned about anemia that is not physiologic if a woman in her second trimester has a hemoglobin level less than A.) 10.5 g/dL B.) 11 g/dL C.) 12 g/dL D.) 13 g/dL

A.) 10.5 g/dL

Assessment of a couple experiencing difficulty conceiving usually begins with: A.) A complete history and physical assessment of both partners. B.) Semen analysis. C.) Testing of cervical mucus for LH surge. D.) Postcoital (Sims-Huhner) test.

A.) A complete history and physical assessment of both partners. Assessment for the origin of infertility always begins with the least costly, noninvasive testing first. A complete history and physical assessment of both the male and female recognizes that infertility can have a female or male origin, or both. This assessment will provide clues about which types of tests, if any, would be most appropriate for this couple.

Fetal movement felt by the examiner is considered a positive indication of pregnancy. However, fetal movement felt by the pregnant woman is considered: A.) A presumptive indication of pregnancy. B.) A probable indication of pregnancy. C.) Also a positive indication of pregnancy.

A.) A presumptive indication of pregnancy. The woman may misinterpret peristalsis in the large intestine for fetal movement. It is unlikely for an experienced examiner to be deceived by these intestinal movements.

Choose appropriate client teaching related to maternal serum alpha-fetoprotein (MSAFP) analysis. A.) Abnormal MSAFP levels should be followed by more specific tests. B.) High MSAFP levels are usually associated with chromosome abnormalities. C.) Having MSAFP testing eliminates the need to perform an ultrasound examination. D.) The inital MSAFP testing will be performed at 12 weeks of gestation.

A.) Abnormal MSAFP levels should be followed by more specific tests.

When teaching a 22-year-old woman about breast self-awareness, the nurse should tell the woman to perform the assessment: A.) About 1 week following the onset of menses. B.) About halfway between menses. C.) On the same day every month. D.) About 1 week before the onset of menses.

A.) About 1 week following the onset of menses. Women should perform a breast self-examination breast self-awareness (BSE) about 1 week following the onset of menses, when hormonal influences on the breasts are at a low level. If the woman no longer menstruates, she may choose a day that is easy to remember and perform the examination on that day every month.

In preparing a pregnant client for a nonstress test, the nurse should: A.) Ask the client if she smokes and when she had her last cigarette. B.) Have the client drink four glasses of water. C.) Ask the client when was the last time she had anything to eat or drink. D.) Ask the client for a urine sample to check for glucose levels.

A.) Ask the client if she smokes and when she had her last cigarette. For the greatest accuracy of a nonstress test, the woman should not have smoked recently because smoking affects the perception of fetal movement. In addition, smoking causes vasoconstriction, which may alter the results.

A laboring client's membranes spontaneously rupture. What should be the nurse's first action? A.) Assess the fetal heart rate. B.) Encourage the woman to ambulate. C.) Document the color, odor, and amount of amniotic fluid. D.) Prepare for imminent delivery of the newborn.

A.) Assess the fetal heart rate.

Which of the following are important points when teaching a client the proper method for pushing during the second stage of labor? (Select all that apply). A.) Begin and end by taking a deep breath and exhaling. B.) Push for 4 to 6 seconds at a time. C.) Take a deep breath and then push while holding her breath. D.) Push at least five or six times with each contraction.

A.) Begin and end by taking a deep breath and exhaling. B.) Push for 4 to 6 seconds at a time.

What would the prenatal clinic nurse conclude to be a contraindication for maternal serum alpha-fetoprotein testing for a pregnant client? A.) Being at 25 weeks' gestation B.) Client would not consider termination of pregnancy C.) Client has no family history of neural tube defects D.) Client had ultrasound at 8 weeks' gestation

A.) Being at 25 weeks' gestation Rationale: This test, which measures the level of maternal serum alpha-fetoprotein, is most sensitive between 16 and 18 weeks' gestation. However, it can be performed at up to 22 weeks' gestation.

To obtain an accurate blood pressure of a woman in labor, the nurse should assess the blood pressure: A.) Between contractions, with the woman lying on her side. B.) Between contractions, with the woman lying on her back. C.) With a contraction while the woman is lying on her side. D.) With a contraction while the woman is lying on her back.

A.) Between contractions, with the woman lying on her side. During uterine contractions, blood flow to the placenta gradually decreases, causing a relative increase in the woman's blood volume. This temporary change increases her blood pressure slightly. If the woman lies on her back, the weight of the fetus, placenta, and fluid may decrease blood flow, causing supine hypotension. Therefore her blood pressure is more accurate when taken between contractions, with her lying on her side.

Which topics are typically addressed in later pregnancy classes? (Select all that apply). A.) Breastfeeding B.) Adapting to pregnancy C.) Childbirth preparation D.) Tips for a healthy pregnancy E.) Parenting skills

A.) Breastfeeding C.) Childbirth preparation E.) Parenting skills

A multigravida at 37 weeks of gestation is admitted to the labor room. She has contractions every 3 to 4 minutes lasting 40 to 50 seconds and no history of clear fluid leakage from the vagina, but complains of bright red bleeding for the past hour. The fetal heart rate is 145 beats/min (bpm). What should be the nurse's next intervention? A.) Call the physician. B.) Perform a vaginal exam to determine imminence of birth. C.) Continue to monitor contractions and fetal heart rate. D.) Administer an enema according to protocol of the agency.

A.) Call the physician. Bright red bleeding is a sign of complications, and the physician or primary health care provider should be notified immediately. Vaginal exams or enemas are contraindicated in the presence of bleeding. Continuing to monitor the mother and fetus is important after notifying the health care provider.

Select which nutrients need to be increased more during a multifetal pregnancy than with a single-fetal pregnancy. (Select all that apply). A.) Calories B.) Iron C.) Calcium D.) Folic acid E.) Sodium

A.) Calories B.) Iron C.) Calcium D.) Folic acid

A newly pregnant woman is trying to choose a health care provider for her pregnancy and birth. She desires to have the health care provider to care for her during the pregnancy, be with her during the labor process, deliver the baby, and care for her and the baby afterward. The best choice for her would be a(n): A.) Certified nurse-midwife. B.) Nurse practitioner. C.) Obstetrician. D.) Family practice physician.

A.) Certified nurse-midwife. The certified nurse-midwife cares for women at low risk for complications during pregnancy, is present during the labor, delivers the infant, and cares for the postpartum mother and infant. The nurse practitioner does not perform deliveries. The obstetrician usually does not care for the woman during labor and does not care for the newborn. The family practice physician usually is not present during most of the labor process.

Which of the following are considered theories about the onset of labor? (Select all that apply). A.) Changes in the relative effects of estrogen and progesterone B.) An increase in prostaglandins C.) Increased secretion of prolactin D.) Decreased secretion of oxytocin E.) Stretching and irritation of the uterus and cervix

A.) Changes in the relative effects of estrogen and progesterone B.) An increase in prostaglandins E.) Stretching and irritation of the uterus and cervix

During the nursing assessment at a routine prenatal visit, the woman's blood pressure was significantly higher than the last visit. Because of this finding, the nurse should: A.) Check the urine for presence of protein. B.) Check the pulse for significant decrease. C.) Assess for pedal edema. D.) Check the urine for presence of glucose.

A.) Check the urine for presence of protein. The two classic signs of preeclampsia are hypertension and proteinuria. Edema in the feet is common in most pregnancies. Edema with preeclampsia may continue up the legs to the hands and face.

When doing a nutritional analysis for a pregnant client, the nurse discovers that she is lacking in vitamin C and folic acid. Which of the following meal suggestions would give the client the most vitamin C and folic acid? A.) Chef's salad with greens, tomatoes, green peppers, spinach, turkey, and Swiss cheese, skim milk, whole wheat crackers B.) Baked chicken, green peas, baked potato, roll, ice tea C.) Grilled hamburger patty with American cheese, whole wheat bun, mustard, ketchup, carrot salad, skim milk D.) Egg omelet with cheese, onions, and ham, apple and raisin salad, ice tea.

A.) Chef's salad with greens, tomatoes, green peppers, spinach, turkey, and Swiss cheese, skim milk, whole wheat crackers The green leafy vegetables, such as salad greens and spinach, are rich in folic acid and also contain vitamin C. The tomatoes and green peppers are rich in vitamin C.

A client who has experienced a spontaneous abortion at 8 weeks asks the nurse why this happened. What would the nurse include in a response to address the most common cause of "miscarriage?" A.) Chromosome abnormalities B.) Environmental teratogens C.) Excessive activity D.) Substance abuse

A.) Chromosome abnormalities Rationale: The majority of early abortions are related to abnormal chromosomes. The client might fear that she has caused the loss, and should be provided with accurate information.

During the labor process, the client's membranes rupture. Select all the assessments that are necessary for the nurse to carry out at this time. A.) Color of amniotic fluid B.) Odor of amniotic fluid C.) Fetal heart rate D.) Cervical dilatation E.) Cervical effacement F.) Time the membranes ruptured

A.) Color of amniotic fluid B.) Odor of amniotic fluid C.) Fetal heart rate F.) Time the membranes ruptured

After several mild late decelerations, the physician obtains a fetal scalp blood sample. The fetal pH was 7.32. The nurse is aware that the next action will probably be to: A.) Continue to monitor the fetus during the labor. B.) Prepare for a cesarean section. C.) Prepare for a reassessment of the fetal pH. D.) Apply oxygen to the mother at a rate of 10 L/min.

A.) Continue to monitor the fetus during the labor. The normal fetal pH is 7.25 to 7.35. Because this fetal pH is within the normal limits, the nurse can anticipate continuing monitoring of the fetus. Because the pH is within normal limits, it is unlikely that a cesarean section would be performed at this time.

Labor pain management may include which of the following interventions? (Select all that apply). A.) Cool, damp washcloths on the face and neck B.) Decreasing bright lights in the room C.) Keeping the woman clean and dry D.) Administering pain medication as ordered E.) Offering simple snacks every 2 hours

A.) Cool, damp washcloths on the face and neck B.) Decreasing bright lights in the room C.) Keeping the woman clean and dry D.) Administering pain medication as ordered

The nurse should include which elements in the psychosocial assessment of the laboring client? Select all that apply. A.) Cultural assessment B.) Plans for naming the child C.) Fetal heart rate assessment D.) Socioeconomic status of the family E.) Expectations of the experience

A.) Cultural assessment E.) Expectations of the experience

The client with preterm labor is being administered magnesium sulfate (generic) intravenously. Which client manifestation is a known side effect of this medication? A.) Decreased deep tendon reflexes B.) Increased respiratory rate C.) Nervousness and tremors D.) Nausea and diarrhea

A.) Decreased deep tendon reflexes Rationale: Magnesium sulfate, when given parenterally, acts as a central nervous system (CNS) depressant and a depressant of smooth, skeletal, and cardiac muscle function. The side effects of this medication when taken IV are drowsiness, flushing, heaviness in the limbs, and decreased deep tendon reflexes.

On review of a fetal monitor tracing, the nurse notes that for several contractions the FHR decelerates as a contraction begins and returns to baseline just before it ends. The nurse should: A.) Describe the finding in the notes. B.) Reposition the woman onto her side. C.) Call the physician for instructions. D.) Administer oxygen at 8 to10 L/min with a tight face mask.

A.) Describe the finding in the notes. An early deceleration pattern from head compression is described. No action other than documentation of the finding is required because this is an expected reaction to compression of the fetal head as it passes through the cervix. The other responses would be implemented when nonreassuring or ominous changes are noted.

The nurse conducting a prenatal education session includes information about danger signs to report to the health care provider. What signs would the nurse include in this teaching session? A.) Dizziness and blurred vision B.) Occasional nausea and vomiting C.) No bowel movement for 3 days D.) Ankle edema

A.) Dizziness and blurred vision Rationale: Dizziness and blurred vision can be symptoms of preeclampsia, a complication that requires further assessment and medical management.

The nurse would assess the pregnant client with a history of multiple sexual partners for which complication of pregnancy of greatest concern in this situation? A.) Ectopic pregnancy B.) Premature rupture of membranes C.) Preeclampsia D.) Rh-incompatibility

A.) Ectopic pregnancy

Choose which of the following topics are usually included in an individual birth plan. (Select all that apply). A.) Electronic fetal monitoring B.)The physician or nurse midwife to use C.) Types of pain relief preferred D.) The support person who should be admitted to the labor and birth room E.) The nurse that the woman prefers to help babysit for her other children during the labor process

A.) Electronic fetal monitoring C.) Types of pain relief preferred D.) The support person who should be admitted to the labor and birth room

The primiparous client with intact membranes is 4 cm dilated, 90% effaced, -1 station, vertex, and having contractions every 5 minutes that are of moderate intensity and last 45 seconds. What would the nurse include in the care plan at this time? A.) Encourage the client to be up and walking. B.) Encourage the client to stay in bed and rest. C.) Monitor the client's blood pressure every 10 minutes. D.) Promote rest by sending the client's partner out of the room.

A.) Encourage the client to be up and walking. Rationale: Upright positioning and walking facilitate the progress of labor.

The nursing care plan for a client with a prolonged latent phase of labor includes which of the following as a priority measure? A.) Encouraging rest and relaxation through the playing of soft music B.) IV hydration with either lactated Ringer's solution or 5 percent dextrose C.) Continuous internal fetal monitoring of the fetal response to contractions D.) Measuring maternal blood pressure, temperature, and pulse every 15 minutes

A.) Encouraging rest and relaxation through the playing of soft music Rationale: Prolonged latent phase of labor is defined as greater than 20 hours in primigravida women and greater than 14 hours in multigravida women. Encouraging rest and relaxation during this phase will help the client have enough energy to push effectively during the second stage of labor. Music is often used effectively to induce relaxation. Encouraging a well-rested client to ambulate will also facilitate the latent phase.

The client is admitted in early labor. Her support person tells the nurse that the contractions have the following pattern: started 1232, ended 1233; started 1235, ended 1236; started 1239, ended 1240; started 1243, ended 1244. From this information, the nurse determines that the frequency of the contractions is: A.) Every 3 to 4 minutes. B.) Every 2 to 3 minutes. C.) Lasting a minute. D.) Unable to be determined with this information.

A.) Every 3 to 4 minutes. The frequency of a contraction is measured from the beginning of one contraction until the beginning of the next contraction. The contractions started at 1232, 1235, 1239, and 1243. This would put the contractions every 3 to 4 minutes. The duration of the contractions is from the beginning of a contraction until the end of the same contraction. The duration for this pattern would be 1 minute.

A pregnant woman at 36 weeks of gestation is admitted to the high-risk pregnancy unit with hypertension. Assessment findings indicate severe preeclampsia. The nurse should: A.) Expect a maintenance dose of intravenous magnesium sulfate to be about 2 g/hr. B.) Assign the woman to a semiprivate room next to the nurses' station. C.) Offer her a diet high in complex carbohydrates and low in salt and protein. D.) Encourage the woman to maintain a semi-Fowler's position when in bed.

A.) Expect a maintenance dose of intravenous magnesium sulfate to be about 2 g/hr. The woman should be assigned to a private room in a quiet area because she is at risk for seizures, which can be precipitated by any sudden activity or sound. Although her diet should contain complex carbohydrates, protein content should be normal to replace protein losses in urine; low-salt diets are not recommended. While on bed rest, the woman should alternate her position from side to side to facilitate placental and renal perfusion, which will enhance cardiac output and facilitate loss of retained fluid.

Which risk factors would necessitate performing a bone density scan on a woman younger than 65 years? (Select all that apply). A.) Family history of osteoporosis B.) Fall history C.) Active life style D.) Chronic steroid use E.) Normal levels of estrogen F.) Overweight

A.) Family history of osteoporosis B.) Fall history D.) Chronic steroid use

Upon completion of a vaginal examination on a laboring woman, the nurse records: 50%, 6 cm, -1. Which of the following is a correct interpretation of the data? A.) Fetal presenting part is 1 cm above the ischial spines. B.) Effacement is 4 cm from completion . C.) Dilation is 50% completed . D.) Fetus has passed through the ischial spines.

A.) Fetal presenting part is 1 cm above the ischial spines. A station of -1 indicates that the fetal presenting part is above the ischial spines and has not yet passed through the pelvic inlet. A station of 0 would indicate that the presenting part has passed through the inlet and is at the level of the ischial spines or is engaged. Passage through the ischial spines with internal rotation would be indicated by a plus station, such as +1. Progress of effacement is referred to by percentages, with 100% indicating full effacement, and dilation by centimeters (cm), with 10 cm indicating full dilation.

The client has a biophysical profile done. The nurse notes that the NST test was reactive, the fetal breathing movements were absent, there was one gross body movement in 30 minutes, the fetal tone showed fetal extremity extension and return to flexion, and one pocket of amniotic fluid measured 3 cm. The nurse anticipates that next action by the physician will be: A.) Further study because a score of 6 is not normal. B.) No further study because a score of 9 is normal. C.) Further study because a score of 3 is not normal. D.) No further study because a score of 10 is perfect.

A.) Further study because a score of 6 is not normal. A score of 8 to 10 is normal for a biophysical profile. To determine the score, five categories are used. The reactive NST gives a score of 2; absent fetal breathing movements is 0; one gross body movement is 0; the fetal tone of extension and flexion is 2; and one pocket of amniotic fluid is 2 points. This gives a total of 6 points, which is not within normal limits, and further studies are indicated.

Women who delay pregnancy until after the age of 35 are at greater risk for which of the following? (Select all that apply). A.) Genetic disorders B.) Preeclampsia C.) Cesarean birth D.) Large-for-gestational age infants E.) Placenta previa F.) Preterm labor

A.) Genetic disorders B.) Preeclampsia C.) Cesarean birth E.) Placenta previa F.) Preterm labor

Ultrasound examinations may be performed at about 12 to 20 weeks of gestation. The main purpose of this study is to determine: A.) Gestational age and may show some fetal anomalies. B.) The gender of the fetus. C.) Cardiac anomalies. D.) Fetal movement and amniotic fluid amounts.

A.) Gestational age and may show some fetal anomalies. Early in the pregnancy the ultrasound will identify gestational age more accurately than dates and may show some fetal anomalies. The sex of the fetus may be determined at that time, but ultrasounds are not done for that reason. Ultrasounds performed in the third trimester may be done to count fetal movement and determine amniotic fluid amounts.

Select which complications are associated with obesity in pregnant women. (Select all that apply). A.) Gestational diabetes B.) Preeclampsia C.) Cesarean birth D.) Small-for-gestational age neonate E.) Hydramnios

A.) Gestational diabetes B.) Preeclampsia C.) Cesarean birth

Which of the following are considered goals of perinatal education? (Select all that apply). A.) Help parents become knowledgeable consumers. B.) Help parents select proper, culturally oriented names for the new baby. C.) Help parents take an active role in maintaining health during pregnancy and birth. D.) Help parents learn coping techniques to deal with pregnancy, childbirth, and parenting.

A.) Help parents become knowledgeable consumers. C.) Help parents take an active role in maintaining health during pregnancy and birth. D.) Help parents learn coping techniques to deal with pregnancy, childbirth, and parenting.

During active labor, the woman complains about tingling in her hands. The nurse's next action should be to: A.) Help the woman slow down her breathing and breathe into her cupped hands. B.) Assess vital signs for changes. C.) Check cervical dilation. D.) Change the woman's position.

A.) Help the woman slow down her breathing and breathe into her cupped hands. Hyperventilation may occur during active labor as the woman breathes rapidly. She may feel tingling in her hands and feet and dizziness. By having the woman slow her breathing and breathe into her cupped hands, her carbon dioxide levels will return to normal and relieve the symptoms.

Why is a cleansing breath at the beginning and end of contractions important? (Select all that apply). A.) Helps the woman release tension B.) Provides oxygen to reduce myometrial hypoxia C.) Provides a diversional activity for the woman D.) Helps the fetus to release tension

A.) Helps the woman release tension B.) Provides oxygen to reduce myometrial hypoxia

Which of the following women are at higher risk for an ectopic pregnancy? (Select all that apply). A.) History of pelvic infection B.) Had a tubal ligation 1 year ago C.) History of hormonal implants for contraception 4 years ago D.) Conception was by assisted reproduction E.) Use of alcohol during the first 2 weeks of the pregnancy F.) History of intrauterine contraceptive device G.) Had five therapeutic abortions

A.) History of pelvic infection B.) Had a tubal ligation 1 year ago D.) Conception was by assisted reproduction F.) History of intrauterine contraceptive device G.) Had five therapeutic abortions

A client has come into the clinic stating she was raped 1 week ago. She did not report the rape but is now concerned that she may be pregnant. The pregnancy test that is most likely to be used at this time is the: A.) Immunoassay test. B.) Enzyme-linked immunosorbent assay (ELISA). C.) Home pregnancy test. D.) There is no test available that can be accurate this early in a pregnancy.

A.) Immunoassay test. The radioimmunoassay test can be accurate as early as 1 week after conception.

A couple seeking infertility counseling expresses their excitement by stating, "Now at last we can become pregnant. We are so glad to get this process started!" The response by the nurse should be based on the knowledge that: A.) Infertility care does not always lead to a successful pregnancy. B.) A successful pregnancy chance can be increased with intervention. C.) Infertility counseling just looks at the cause of the infertility, not its treatment. D.) A pregnancy can occur, but there is no guarantee about the successful completion of the pregnancy.

A.) Infertility care does not always lead to a successful pregnancy. Because some factors contributing to infertility remain unknown, treatment of an identified problem does not always lead to a successful pregnancy.

How does childbirth pain differs from other types of pain? (Select all that apply). A.) It is a normal process. B.) There is preparation time. C.) It is stronger than most other types of pain. D.) It is self-limiting. E.) It is intermittent. F.) It is always a dull, achy type of pain.

A.) It is a normal process. B.) There is preparation time. D.) It is self-limiting. E.) It is intermittent.

The client is in for her first prenatal visit. She tells the nurse that she had a normal period starting on April 3 and spotted on May 1. Using Nagele's rule, the nurse establishes the estimated date of birth as: A.) January 10. B.) February 8. C.) October 6. D.) November 4.

A.) January 10. Nagele's rule is to take the first day of the last normal menstrual period, subtract 3 months, and then add 7 days. Spotting may occur during the time of the first missed period when the embryo implants.

Choose the abbreviation that represents the fetal presentation and position that is most favorable for vaginal birth. A.) LOA B.) RMP C.) LST D.) ROP

A.) LOA

The nurse notes that earlier in the shift, the fetal heart rate (FHR) was 140 beats/minute and the baseline has now risen to 170. The nurse would investigate which factors as possible causes for the higher baseline readings? Select all that apply. A.) Maternal fever B.) Narcotic administration C.) Fetal movement D.) Utero-placental insufficiency E.) Fetal distress

A.) Maternal fever E.) Fetal distress

When comparing the uterine souffle to the funic souffle, the uterine souffle would have the same rate as the: A.) Maternal heart rate. B.) Fetal heart rate. C.) Respiratory rate.

A.) Maternal heart rate. The uterine souffle is a soft blowing sound auscultated over the uterus. It corresponds to the maternal pulse. The funic souffle is heard over the umbilical cord and corresponds to the fetal heart rate.

Which of these might cause late decelerations in the fetus? (Select all that apply). A.) Maternal hypotension B.) Excessive uterine activity C.) Maternal hypertension D.) Fever E.) Maternal overhydration F.) Prolapsed cord

A.) Maternal hypotension B.) Excessive uterine activity C.) Maternal hypertension

Rubin described specific steps that provide a framework for understanding maternal role development. These steps include which of the following? (Select all that apply). A.) Mimicry B.) Fantasy C.) Ambivalence D.) Grief work E.) Behavior modification

A.) Mimicry B.) Fantasy D.) Grief work

A client at 24 weeks' gestation has been scheduled for an amniocentesis. Which actions should the nurse plan to take in the care of this client? Select all that apply. A.) Monitor maternal vital signs and fetal heart rate during procedure. B.) Instruct that an ultrasound machine will be used during the procedure. C.) Assist the woman in assuming a supine position with a wedge under left hip. D.) Have a consent signed for epidural analgesia. E.) Explain that 60 mL of amniotic fluid will be withdrawn.

A.) Monitor maternal vital signs and fetal heart rate during procedure. B.) Instruct that an ultrasound machine will be used during the procedure. C.) Assist the woman in assuming a supine position with a wedge under left hip.

A neonatal nurse is attending a high-risk delivery, and is told that the mother received morphine sulfate IV 30 minutes ago. The nurse should be prepared to give which medication to the infant immediately after delivery? A.) Naloxone (Narcan) B.) Regular insulin C.) Double dose of vitamin K (AquaMEPHYTON) D.) Magnesium sulfate

A.) Naloxone (Narcan) Rationale: Opioid analgesics (narcotics) such as morphine cross the placenta, and can cause respiratory depression in a neonate when given shortly before delivery. Naloxone (Narcan) is the drug of choice to reverse respiratory depression in the neonate caused by narcotics.

A pregnant woman has a glucose challenge test at 26 weeks of gestation. The result is 128 mg/dL. The nurse should expect that: A.) No additional glucose testing will be needed. B.) Insulin injections will be needed by 30 weeks of gestation. C.) Oral drugs may be prescribed to lower her glucose level. D.) More testing is needed to determine appropriate therapy.

A.) No additional glucose testing will be needed.

To ensure adequate fetal oxygenation, which of the following are needed? (Select all that apply). A.) Normal maternal blood flow and volume to the placenta B.) Normal oxygen saturation in maternal blood C.) Normal carbon dioxide saturation in the maternal blood D.) Adequate exchange of oxygen and carbon dioxide in the placenta E.) Normal fetal circulatory and oxygen-carrying functions F.) Normal blood glucose levels in the fetal circulation

A.) Normal maternal blood flow and volume to the placenta B.) Normal oxygen saturation in maternal blood D.) Adequate exchange of oxygen and carbon dioxide in the placenta E.) Normal fetal circulatory and oxygen-carrying functions

A woman is admitted with a diagnosis of missed abortion. After taking her blood pressure, the nurse notices petechiae on the woman's arm where the cuff was located. The nurse's next action should be to: A.) Notify the health care provider. B.) Massage the arm. C.) Monitor her blood pressure closely. D.) Determine her temperature.

A.) Notify the health care provider. One major complication of missed abortion is disseminated intravascular coagulation (DIC). This may be manifested by small areas of hemorrhaging. The health care provider needs to be notified.

Which of the following should be included in nursing care during labor? (Select all that apply). A.) Offer ice chips in small amounts to relieve a dry mouth. B.) Monitor for a full bladder because the woman may have a decreased sensation of the urge to void. C.) Keep the woman in a side-lying position to prevent supine hypotension. D.) Offer small bland meals if the woman is in early labor to help maintain proper blood sugar levels. E.) Monitor the fetal heart rate for changes from normal.

A.) Offer ice chips in small amounts to relieve a dry mouth. B.) Monitor for a full bladder because the woman may have a decreased sensation of the urge to void. C.) Keep the woman in a side-lying position to prevent supine hypotension. E.) Monitor the fetal heart rate for changes from normal.

After a planned cesarean section, the woman is being admitted back to the postpartum unit. The nurse notices that the client is rubbing her nose and eyes continually. Being aware that the woman has been given epidural opioids, the nurse's next action should be to: A.) Offer the woman some medication to relieve the itching. B.) Notify the anesthesiologist immediately. C.) Monitor for signs of respiratory depression. D.) Monitor the client's temperature.

A.) Offer the woman some medication to relieve the itching. Pruritus of the face and neck is an annoying side effect that may occur with epidural opioids. Medications may be used to relieve the itching and make the woman more comfortable.

Which conditions are associated with elevated levels of serum alpha-fetoprotein? (Select all that apply). A.) Open neural tube defects B.) Threatened abortion C.) Chromosomal trisomies D.) Gestational trophoblastic disease E.) Fetal demise

A.) Open neural tube defects B.) Threatened abortion E.) Fetal demise

The primary function of human chorionic gonadotropin during pregnancy is to: A.) Prevent deterioration of the corpus luteum. B.) Produce the extra amount of estrogen and progesterone that is needed. C.) Prevent release of the luteinizing hormone. D.) Stimulate uterine growth.

A.) Prevent deterioration of the corpus luteum. Human chorionic gonadotropin is produced by the trophoblastic cells and prevents deterioration of the corpus luteum. The corpus luteum produces the progesterone needed for the pregnancy until the placenta is developed.

The nurse should tell a primigravida that the definitive sign indicating labor has begun is: A.) Progressive uterine contractions. B.) Lightening. C.) Rupture of membranes. D.) Passage of the mucus plug.

A.) Progressive uterine contractions. Regular, progressive uterine contractions that increase in intensity and frequency are a sign of true labor. Responses B and D are premonitory signs indicating that the onset of labor is getting closer. Rupture of membranes usually occurs during labor itself.

Relaxation of the mother during labor is important for several reasons. Listed below are the reasons that promoting relaxation is important. (Select all that apply). A.) Promotes uterine blood flow B.) Improves fetal oxygenation C.) Promotes efficient uterine contractions D.) Reduces tension that increases pain E.) Inhibits rapid fetal descent

A.) Promotes uterine blood flow B.) Improves fetal oxygenation C.) Promotes efficient uterine contractions D.) Reduces tension that increases pain

The nurse notices that during the nonstress test, the fetal heart rate accelerated 20 beats/min above baseline three times with fetal movement. Each acceleration lasted about 15 seconds before returning to baseline. This result is classified as: A.) Reactive. B.) Nonreactive.

A.) Reactive. A reactive (reassuring) nonstress test shows at least two fetal heart accelerations, with or without fetal movement, within a 20-minute period. The accelerations peak at least 15 beats/min above baseline and last 15 seconds from baseline to baseline.

During a vaginal exam, the physician stimulates the fetal scalp. The fetal heart rate accelerated from 140 to 155 bpm for about 30 seconds. The nurse should: A.) Record this reassuring fetal reaction. B.) Notify the physician because this reaction is nonreassuring. C.) Assist the woman into a side-lying position. D.) Administer oxygen at 8 to 10 L/min.

A.) Record this reassuring fetal reaction. It is reassuring for the heart rate to elevate 15 bpm for at least 15 seconds with fetal scalp stimulation. The nurse should record the finding. No other intervention is necessary at this time.

The nurse notes a pattern of variable decelerations to 75 bpm on the fetal monitor. The initial nursing action should be to: A.) Reposition the woman. B.) Administer oxygen. C.) Increase the intravenous (IV) infusion. D.) Stimulate the fetal scalp.

A.) Reposition the woman.

The client has refused sedation ordered by the physician for hypertonic contractions and prolonged latent-phase labor for fear that her labor will stop. The nurse can help by explaining which of the following? A.) Sedation helps to provide needed rest, and allows time for the uterine contractions to become coordinated, so that labor is progressive. B.) If the woman is experiencing true labor, contractions will not stop even with sedation. C.) If contractions continue without cervical effacement and dilatation, the fetus is at risk for hypoxia. D.) Sedation will stop contractions that are uncoordinated, and will provide more time to determine if a cesarean delivery is needed.

A.) Sedation helps to provide needed rest, and allows time for the uterine contractions to become coordinated, so that labor is progressive.

During pregnancy, the cardiac output is highest when the woman is in which position? A.) Side-lying B.) Sitting C.) Standing D.) Supine

A.) Side-lying Cardiac output is highest when the woman is lying on her side and lower in the sitting, standing, or supine position.

Variability can be reduced by which of the following factors? (Select all that apply). A.) Sleep B.) Narcotics C.) Gestation longer than 39 weeks D.) Fetal anomalies that affect the central nervous system

A.) Sleep B.) Narcotics D.) Fetal anomalies that affect the central nervous system

Which of the following are considered nonreassuring fetal heart rate patterns? (Select all that apply). A.) Tachycardia B.) Bradycardia C.) Absent variability D.) Early decelerations E.) Variable decelerations

A.) Tachycardia B.) Bradycardia C.) Absent variability E.) Variable decelerations

Clonus indicates which of the following: A.) The central nervous system is very irritable. B.) Renal blood flow is severely reduced. C.) Lungs are filling with interstitial fluid. D.) Muscles of the foot are inflamed.

A.) The central nervous system is very irritable.

Which of the following is correct concerning the performance of a Pap test? A.) The woman should not douche, use vaginal medications, or have intercourse for at least 24 hours before the test. B.) It should be performed once a year, beginning with the onset of puberty. C.) A lubricant such as petroleum jelly should be used to ease speculum insertion. D.) The specimen for the Pap test should be obtained after a specimen is collected to test for cervical infection.

A.) The woman should not douche, use vaginal medications, or have intercourse for at least 24 hours before the test. Pap tests are performed annually for sexually active women or by age 18, especially if risk factors for cervical cancer or reproductive tract infections are present. Pap tests may be performed every 3 years in low-risk women after three negative results on annual tests have been obtained. No lubricant other than warm water should be used because accuracy of the test can be affected. The cytologic specimen should be obtained first.

Which factors contribute to pain during labor and birth? (Select all that apply). A.) Tissue ischemia B.) Cutting of the nerves with dilatation C.) Cervical dilation D.) Distention of the vagina and perineum

A.) Tissue ischemia C.) Cervical dilation D.) Distention of the vagina and perineum

An intravenous access is started in most labor clients because of which of the following? (Select all that apply). A.) To have quick access if drugs are needed B.) To provide fluids to prevent dehydration C.) In case an epidural block is administered D.) To provide a route for pain medications for the 48-hour postpartum period

A.) To have quick access if drugs are needed B.) To provide fluids to prevent dehydration C.) In case an epidural block is administered

Select the reasons that a preconception visit is important for a woman. (Select all that apply). A.) To identify preexisting problems that may affect the fetus B.) To identify medications the woman is taking that may be harmful to the fetus C.) To inform her to start increasing her folic acid intake D.) To develop a plan of care for the pregnancy E.) To determine the best time of the year to conceive

A.) To identify preexisting problems that may affect the fetus B.) To identify medications the woman is taking that may be harmful to the fetus C.) To inform her to start increasing her folic acid intake

Caffeine intake for a lactating mother should be limited to: A.) Two cups of coffee a day. B.) Three cups of coffee a day. C.) Four cups of coffee a day. D.) Five cups of coffee a day.

A.) Two cups of coffee a day. The mother should restrict her caffeine intake to two cups of coffee or the equivalent each day. Caffeine in excessive amounts can make some infants irritable.

A pregnant woman should be taught that the first sign of a threatened abortion is usually: A.) Vaginal bleeding. B.) Uterine cramping. C.) Rupture of membranes. D.) Backache.

A.) Vaginal bleeding. The first sign of threatened abortion is vaginal bleeding, which is rather common during early pregnancy. One third of pregnant women experience bleeding in early pregnancy and up to 50% of these pregnancies end in A spontaneous abortion. The vaginal bleeding may be followed by uterine cramping and backache.

With regard to normal changes in the reproductive system during pregnancy, the nurse should teach the pregnant client about which of the following? A.) Vaginal secretions will increase and thicken. B.) Uterus will grow by adding many new cells. C.) Breasts will become red and hard. D.) Cervix will begin to dilate during the second trimester.

A.) Vaginal secretions will increase and thicken.

Name the four periodic changes of the FHR, their causes, and one nursing treatment for each.

Accelerations are caused by a burst of sympathetic activity; they are reassuring and require no treatment. Early decelerations are caused by head compression; they are benign and alert the nurse to monitor for labor progress and fetal descent. Variable decelerations are caused by cord compression; change of position should be tried first. Late decelerations are caused by UPI and should be treated by placing client on her side and administering oxygen.

Name five maternal variables associated with diagnosis of a high-risk pregnancy.

Age (under 17 or over 34 years of age); parity (over 5); <3 months between pregnancies; diagnosis of preeclampsia, diabetes mellitus, or cardiac disease.

State the normal psychosocial responses to pregnancy in the second trimester.

Ambivalence wanes and acceptance of pregnancy occurs; pregnancy becomes "real"; signs of maternal-fetal bonding occur.

Which statement by the pregnant client in her first trimester indicates that she understood prenatal education? A.) "As long as I don't drink more than one serving of alcohol per day, my baby will not be harmed." B.) "Caffeine won't harm my baby, but it might decrease my iron absorption." C.) "Tobacco is only a danger to my baby if I smoke in the first trimester." D.) "Research has shown that if I use marijuana during pregnancy, I will probably have a baby with birth defects."

B.) "Caffeine won't harm my baby, but it might decrease my iron absorption." Rationale: Caffeine has been found to interfere with absorption of iron.

The nurse is admitting a client who is in early labor. The client presents a written document to the nurse stating that this is my "birth plan" that will be carried out while I am here. How can nurse can best respond to the client? (Select all that apply). A.) "We will comply with all of your wishes." B.) "Let's go over this list together and discuss each item." C.) "We will not be able to comply with these wishes." D.) "We will try to comply with as many as possible; however, as the labor progresses, complications may necessitate changes in the plan."

B.) "Let's go over this list together and discuss each item." D.) "We will try to comply with as many as possible; however, as the labor progresses, complications may necessitate changes in the plan." The client and staff need to work together on a plan that will be appropriate for labor, birth, and postpartum care. The nurse should not promise to comply with the client's wishes when complications may necessitate changes in the plan. Similarly, the nurse should not deny the client's wishes if they are medically safe and can be accommodated.

Which of the following statements by a woman shows she is trying to confirm the pregnancy? A.) "I feel good, full of energy." B.) "My jeans are getting too tight." C.) "My periods have always been irregular." D.) "I have had some nausea, but the flu is going around at work."

B.) "My jeans are getting too tight." When a woman is trying to confirm the pregnancy, she observes her body carefully for changes, such as an enlarging abdomen. The other answers show she is trying to deny the pregnancy.

The nurse concludes that a client is at risk for preeclampsia when the vital signs taken today show that the blood pressure has shown which pattern of elevation since the previous prenatal visit? A.) 90/56 to 110/70 B.) 100/60 to 130/76 C.) 122/80 to 138/86 D.) 134/80 to 140/88

B.) 100/60 to 130/76

A woman is admitted to the maternity unit with preeclampsia. She is started on magnesium sulfate IV, a urinary catheter is inserted, and she is put on bed rest. The nurse understands it is important to monitor urinary output hourly. It is important that the client have an output of at least: A.) 15 mL. B.) 30 mL. C.) 60 mL. D.) 100 mL.

B.) 30 mL. Normal renal profusion should be an hourly output of at least 30 mL. Decreased urinary output indicates poor perfusion of the kidneys and may precede acute renal failure.

An 8-month-pregnant woman should increase her calorie intake by: A.) 300 calories over her prepregnancy needs. B.) 452 calories over her prepregnancy needs. C.) 550 calories over her prepregnancy needs. D.) 300 calories per baby over her prepregnancy needs.

B.) 452 calories over her prepregnancy needs. The caloric intake for adolescent and adult pregnant women should increase by 340 calories during the second trimester and 452 calories during the third trimester. She does not need to increase calories during the first trimester.

After birth, the nurse assesses the newborn. The heart rate is 90 bpm, the body is flexed, there is vigorous movement, the newborn is actively crying when stimulated, and has bluish coloration in the feet and hands. The proper Apgar score for this newborn should be: A.) 7. B.) 8. C.) 9. D.) 10.

B.) 8. The heart rate less than 100 bpm gets a score of 1, a lusty cry will give a score of 2 for both respiratory effort and reflex response, the flexed posture and vigorous movements gives a score of 2, and the bluish coloration of the hands and feet will give a score of 1.

A nurse who deals primarily with teenagers who are pregnant is aware that the pregnancy will cause which of the following? (Select all that apply). A.) A slowing of the developmental stages for the mother B.) A financial burden that is difficult for teenagers C.) Emotional upheaval as they make decisions about their future and the future of the child D.) Higher risk factors for the mother and infant

B.) A financial burden that is difficult for teenagers C.) Emotional upheaval as they make decisions about their future and the future of the child D.) Higher risk factors for the mother and infant

Which of the following women can the nurse anticipate having difficulty dealing with labor pain? A.) Primigravida who has attended childbirth preparation classes B.) A woman having her second baby; the first child was in a posterior position and the labor lasted 18 hours. C.) A woman having her sixth child and who has not attended any prenatal teaching classes D.) Primigravida who has her mother as her birth support person. The mother is encouraging her with every contraction.

B.) A woman having her second baby; the first child was in a posterior position and the labor lasted 18 hours. Previous experiences with pain can alter a woman's perception of labor pain. The woman with a prolonged labor and posterior position with the last birth will come to this labor anxious about the outcome and amount of pain. Preparation for labor and previous positive experiences will help the woman tolerate the pain. A support person who has been through the process and is encouraging can also assist the woman in a positive way.

A woman in labor has a long history of uncontrolled hypertension. The hypertension has continued throughout the pregnancy and labor. The nurse is aware that the woman is at high risk for which complication? A.) Placenta previa B.) Abruptio placentae C.) Hypotonic contractions D.) DIC

B.) Abruptio placentae Risk factors for abruptio placentae include maternal hypertension. Vasoconstriction is an effect of hypertension that can affect the endometrial arteries.

The nurse would formulate which wellness-oriented nursing diagnosis for a client in the second trimester of pregnancy? A.) Anxiety related to lack of understanding about early prenatal physical changes B.) Beginning Maternal-Fetal Attachment related to statements about perception of fetal movement C.) Promoting Client Safety related to falls prevention D.) Knowledge Deficit related to lack of preparation for labor and delivery

B.) Beginning Maternal-Fetal Attachment related to statements about perception of fetal movement Rationale: Quickening usually begins around 16 weeks and results in enhanced attachment as the fetus becomes more real.

During the initial assessment of a 3-month-pregnant client, the nurse discovers that the client has to stand up for 8 hours at work. The nurse is aware that this may increase the client's risk of: A.) Ankle edema. B.) Blood clots. C.) Preterm labor. D.) Vaginal bleeding.

B.) Blood clots. During pregnancy, the plasma fibrinogen level rises about 50% and plasma fibrin level increases about 40%. These increase the risk of blood clots. The fact that the client stands for prolonged periods increases stasis of blood in the veins of the legs, which further increases the risk for blood clots.

A 39-week primigravida calls the birthing center and tells the nurse she has contractions that are 10 to 15 minutes apart and had a small gush of fluid about 1 hour ago. The nurse should tell her to: A.) Wait until the contractions are about 5 minutes apart and come to the center. B.) Come to the birthing center now. C.) Come to the birthing center in about an hour if she lives farther than 1 hour away. D.) Come to the birthing center if the baby stops moving.

B.) Come to the birthing center now. A gush or trickle of fluid from the vagina should be evaluated as soon as possible. Waiting until the contractions are 5 minutes apart is appropriate for a primigravida if the membranes have not ruptured.

After a percutaneous umbilical blood sampling (PUBS) on a client, the nurse notes a fetal heart rate of 100 beats/min. The nurse is aware that this indicates a: A.) Major complication, and interventions are necessary immediately. B.) Common complication and is usually brief. C.) Minor complication that needs to be monitored for the next 2 weeks

B.) Common complication and is usually brief. Fetal bradycardia is the most common complication after PUBS, is usually brief, and has no long-term consequences.

Changes in the respiratory system that may be caused by progesterone include which of the following? (Select all that apply). A.) Hypoventilation B.) Decrease in airway resistance by relaxing the smooth muscles C.) Increase in the sensitivity of the respiratory center to carbon dioxide

B.) Decrease in airway resistance by relaxing the smooth muscles C.) Increase in the sensitivity of the respiratory center to carbon dioxide

Which of the following characteristics is associated with false labor contractions? A.) Painless B.) Decrease in intensity with ambulation C.) Regular pattern of frequency is established D.) Progressive in terms of intensity and duration

B.) Decrease in intensity with ambulation False labor contractions decrease with activity, but true labor contractions are enhanced or stimulated with activity such as ambulation. False labor contractions are painful. Responses C and D are characteristics of true labor contractions, which increase in intensity with activities such as ambulation.

A pregnant client is receiving magnesium sulfate. What should the nurse evaluate as a sign of excessive blood levels of the drug? A.) Development of seizures B.) Disappearance of the knee-jerk reflex C.) Increase in respiratory rate D.) Increase in blood pressure

B.) Disappearance of the knee-jerk reflex

A woman who is 3 months pregnant desires to attend a class to assist her in dealing with the physical changes and discomforts she is experiencing. The nurse should recommend a(n): A.) Preconception class. B.) Early pregnancy class. C.) Childbirth preparation class. D.) Parenting class.

B.) Early pregnancy class. Early pregnancy classes focus on the first two trimesters. They cover information on adapting to pregnancy, dealing with early discomforts such as morning sickness and fatigue, and understanding what to expect in the months ahead. Preconception classes are for couples who are thinking about having a baby. Childbirth preparation classes teach self-help measures and what to expect for labor and birth. Parenting classes teach newborn care.

Nursing teaching for a woman who has hyperemesis gravidarum should include which of the following? A.) Adding favorite seasonings to foods while cooking B.) Eating simple foods, such as breads and fruits C.) Lying down on her right side after eating D.) Eating creamed soup with every meal

B.) Eating simple foods, such as breads and fruits

In caring for a low-risk woman in the active phase of labor, the nurse realizes the assessment of fetal well-being should occur: A.) Every 15 minutes. B.) Every 30 minutes. C.) Every 5 minutes. D.) Every hour.

B.) Every 30 minutes. For low-risk women, the nurse should evaluate the fetal monitoring strip or assessment fetal well-being at least every 30 minutes during the active phase of labor and every 15 minutes during the second stage. For the high-risk woman, monitoring should occur every 15 minutes during the active phase and every 5 minutes during the second stage.

A woman who is assessing fetal movements each day should notify her health care provider if: A.) More than six movements are felt during a 30-to 60- minute period. B.) Fetal movements are fewer than the minimum set by the provider. C.) The movement pattern remains about the same from day to day. D.) Fetal movements are more frequent during the evening than in the morning.

B.) Fetal movements are fewer than the minimum set by the provider.

Firm sacral pressure is likely to be most helpful in which situation? A.) Rapid labor and birth B.) Fetal occiput posterior position C.) Oxytocin induction of labor D.) If analgesics should be avoided

B.) Fetal occiput posterior position A posterior position of the vertex will cause pressure against the sacrum. This pressure increases back pain during and between contractions. Firm sacral pressure may help relieve some of the pressure.

During each contraction, the nurse notices that the woman stops talking and stares at a picture on the wall. The nurse realizes that the woman is using the picture as a: A.) Point of imagery. B.) Focal point. C.) Distraction.

B.) Focal point. The focal point is an object on which the woman centers her attention during contractions. It helps her direct her thoughts away from the contractions. Imagery is a technique for relaxation when the woman imagines specific scenes that are relaxing. Distraction can be used in the early phase of labor. The woman concentrates on something else, such as playing cards or watching a favorite movie.

Which of the following findings during the fourth stage would require immediate interventions by the nurse? A.) Fundus firm and at midline B.) Fundus firm, deviated to the right, with slight distention over the symphysis pubis C.) Blood pressure and pulse slightly lower than reading during second stage of labor D.) Lochia is bright red, with a few small clots

B.) Fundus firm, deviated to the right, with slight distention over the symphysis pubis Even though the fundus is firm, it is not midline and the bladder is filling. A full bladder will interfere with contractions of the uterus and lead to increased bleeding. The rest of the answer choices are within normal limits for this stage.

During the latent phase of labor, the nurse suggests that the woman play cards with her husband. The nurse is aware that this will help the woman deal with the pain of contractions. This technique is called: A.) Cutaneous stimulation. B.) Gate control theory. C.) Thermal stimulation. D.) Hydrotherapy.

B.) Gate control theory. In the gate control theory of pain, the use of cognitive processes can affect the perception of stimuli as painful. Diversional activities in early labor and focal points or breathing techniques later in labor are examples of the gate control theory of pain. Cutaneous stimulation is using touch to relax the muscles. Thermal stimulation is the use of warmth to relax the muscles. Hydrotherapy is the use of water for relaxation.

The nurse notices on the admission record that the fetus is in a cephalic military presentation. The nurse realizes that the fetus: A.) Is coming feet first into the birth canal. B.) Has the head in the birth canal first, but the head is not flexed. C.) Has the head in the birth canal first, and the head is in a flexed presentation. D.) Has both feet coming into the birth canal first.

B.) Has the head in the birth canal first, but the head is not flexed. Cephalic presentation shows that the head is coming into the birth canal first. The military presentation means that the head is in a neutral position, neither flexed nor extended.

A couple delivered a full-term baby girl 3 years ago. They have been attempting pregnancy for the past 2 years, without success. When taking a history from the couple, which of the following may lead to abnormal sperm numbers and function in the male? A.) His job requires him to walk most of the day, with few rest periods. B.) He was infected with mumps 2 years ago after the birth of his girl. C.) He was diagnosed with hypertension 2 years ago and is under medical treatment. D.) His job requires him to be outside about 7 hours out of the 8 hours at work.

B.) He was infected with mumps 2 years ago after the birth of his girl. Acute or chronic illness such as mumps, cirrhosis, and renal failure can impair the number and function of the sperm. Sitting for prolonged periods will elevate the scrotal temperature and impair the numbers and function of the sperm. Antihypertensives may produce abnormal ejaculation but do not interfere with the sperm number and function.

The clinic nurse is interviewing a client couple for an initial infertility workup. Which priority topic would the nurse plan to address? A.) Whether or not the couple has medical insurance B.) How infertility is affecting their lives C.) Whether the man has seafood allergies D.) Whether the woman works outside the home

B.) How infertility is affecting their lives

A client with preeclampsia is receiving magnesium sulfate. For what effect should the nurse observe the client during administration of the drug? A.) Dry, pale skin B.) Hyporeflexia C.) Agitation D.) Increased respirations

B.) Hyporeflexia

The abbreviation LOA means that the fetal occiput is: A.) On the examiner's left and in the front of the pelvis. B.) In the left front part of the mother's pelvis. C.) Anterior of the fetal breech. D.) Lower than the fetal breech.

B.) In the left front part of the mother's pelvis.

Signs associated with frequent or recent use of crack cocaine in the mother include which of the following? (Select all that apply). A.) Pinpoint pupils B.) Increased body temperature C.) Profuse sweating D.) Hypertension

B.) Increased body temperature C.) Profuse sweating D.) Hypertension

After noting that a laboring client is anxious, the maternal newborn nurse anticipates that the anxiety may have which effect on the client? A.) Rapid progression of labor B.) Increased pain during labor C.) No reliance on support person D.) Need for episiotomy

B.) Increased pain during labor Rationale: Anxiety commonly increases the perception of pain, and childbearing is no exception to this. Decreasing anxiety through education and support will facilitate the birthing process.

Which of the following nutrients may be lacking in a lactovegetarian's diet? A.) Calcium B.) Iron C.) Vitamin C D.) Vitamin A

B.) Iron Iron in the vegetarian diet is poorly absorbed because of the lack of heme iron from meats. Iron supplementation is particularly important for vegetarian women during pregnancy. Because lactovegetarians eat dairy products, calcium is adequate. Because there is an adequate intake of fruits and vegetables in a vegetarian's diet, there should be no deficiency of vitamins A and C.

The nurse is admitting 37-week-pregnant woman with severe preeclampsia. When choosing a room for her, the nurse would put her in the: A.) First room near the elevator. B.) Last room at the end of the hall, close to the nurses' station. C.) Room next to the nursery. D.) Room across from the visitors' lounge.

B.) Last room at the end of the hall, close to the nurses' station. The mother needs to be closely monitored. However, she also needs an area that is away from external stimuli and is quiet. All the other rooms would have visitors and noise associated with them.

As the nurse is admitting a woman in labor, she notices that the woman is happy and excited that she is in labor. The contractions are 5 minutes apart, lasting 30 to 35 seconds. The nurse can anticipate that the client is in which phase of labor? A.) Second B.) Latent C.) Active D.) Transition

B.) Latent During the latent phase of the first stage of labor, the woman is usually sociable, excited, and cooperative. The contractions are about 5 minutes apart.

The nurse is preparing to auscultate the fetal heart rate using a Doppler transducer. When performing the Leopold maneuver, the nurse felt the buttocks near the fundus and the back along the left side of the mother. The best position for the Doppler would be in the mother's: A.) Left upper quadrant. B.) Left lower quadrant. C.) Right upper quadrant. D.) Right lower quadrant.

B.) Left lower quadrant. The fetal heart is best heard through the fetus's upper back. Because this fetus is in a cephalic position, with the back toward the mother's left side, the Doppler should be placed in the left lower quadrant of the mother's abdomen.

An ultrasound is done prior to an amniocentesis to: A.) Determine fetal age. B.) Locate fetal and placental position. C.) Determine maternal blood pressure. D.) Determine amount of fetal movement.

B.) Locate fetal and placental position. Determining fetal and placental position is important prior to amniocentesis to prevent damage by the needle.

Select the functions of progesterone during pregnancy. (Select all that apply). A.) Increases blood supply to uterine vessels B.) Maintains the endometrial layer for implantation C.) Relaxes smooth muscles D.) Stimulates uterine growth E.) Facilitates the deposit of maternal fat stores

B.) Maintains the endometrial layer for implantation C.) Relaxes smooth muscles E.) Facilitates the deposit of maternal fat stores

Which nursing action would take priority when caring for the woman with a suspected ectopic pregnancy? A.) Administering oxygen B.) Monitoring vital signs C.) Obtaining surgeon's responsibility D.) Providing emotional support

B.) Monitoring vital signs Rationale: The client with a suspected ectopic pregnancy might be at risk for the development of hypovolemic shock. Assessment is the first step of the nursing process, and airway, breathing, and circulation are the priorities.

As labor progresses, the nurse expects to assess that a pregnant client's contractions have which characteristics? A.) More intense, less frequent, and of longer duration B.) More intense, more frequent, and of longer duration C.) Constant in intensity, more frequent, and of shorter duration D.) Constant in intensity and frequency, but of shorter duration

B.) More intense, more frequent, and of longer duration Rationale: As labor progresses, contractions will become more intense, occur more frequently (shorter resting phase between contractions), and have an increasing duration.

Semen analysis is a common diagnostic procedure related to infertility. When instructing a male client about this test, the nurse would tell him to: A.) Ejaculate into a sterile container. B.) Obtain the specimen after a period of abstinence from ejaculation for 2 to 5 days. C.) Transport the specimen with the container packed in ice. D.) Ensure that the specimen arrives at the laboratory within 30 minutes of ejaculation.

B.) Obtain the specimen after a period of abstinence from ejaculation for 2 to 5 days. The male must ejaculate into a clean container or plastic sheath that does not contain a spermicide. He should avoid exposing the specimen to extremes of temperature, heat or cold. The specimen should be taken to the laboratory within 2 hours of ejaculation.

When scheduling times for women to have a pelvic examination and Papanicolaou (Pap) test, what question is important to ask the woman? A.) When was her last examination? B.) On what date will her next menstrual period start? C.) Does she have insurance coverage of the examination? D.) Does she use any type of birth control?

B.) On what date will her next menstrual period start? Pelvic examinations should be scheduled between menstrual periods.

One comparison between teenage mothers and mature primiparas is that they lack: A.) Parenting skills and have trouble learning them. B.) Peer support because their friends do not have babies. C.) Financial support. D.) Support from the father.

B.) Peer support because their friends do not have babies. Both teenage mothers and mature primiparas find that their normal set of friends do not have infants. The mature mother has friends with teenage children, whereas the teenager has friends who do not have children. They both may lack parenting skills, but both groups can learn them. The mature mother is usually eager to learn as much about childrearing as possible. The mature mother usually does not have the same financial burden as the teenager. The mature mother has more support from the baby's father.

A nurse teaching adolescents concerning care during menses should include that: A.) Only perineal pads should be used until the woman is at least 24 years old to allow closure of the cervical os. B.) Perineal pads should be worn at night. C.) Tampons can be used around the clock. D.) Tampons should be replaced every 6 to 8 hours.

B.) Perineal pads should be worn at night. When using tampons, they should be changed at least every 4 hours to prevent excessive bacterial growth. Perineal pads should be used at night during sleep, which usually exceeds 4 hours.

An ultrasound is ordered for a client who is 8½ months pregnant. It is important for the nurse to: A.) Instruct the client to drink several glasses of clear fluid 1 hour before the examination and not to void. B.) Place a wedge under one hip when placing her on the examination table. C.) Place the client in the lithotomy position for the examination. D.) Instruct the client to bring someone with her to the examination to drive her home.

B.) Place a wedge under one hip when placing her on the examination table. When lying supine, the client is at risk for supine hypotension. Placing a wedge or rolled blanket under one hip will move the gravid uterus over to the side away from the vena cava and aorta. Drinking several glasses of clear fluid is needed during the second trimester prior to an ultrasound. A vaginal ultrasound is done during the first trimester.

During a contraction stress test, the nurse notices late decelerations with three of the six contractions. These results are classified as: A.) Negative. B.) Positive. C.) Equivocal. D.) Unsatisfactory.

B.) Positive. Positive results are when late decelerations accompany at least 50% of the contractions.

Which of the following are consequences of maternal cigarette use during pregnancy? (Select all that apply). A.) Large-for-gestational age infants B.) Prematurity C.) Neurologic development problems D.) Hyperactivity E.) Short attention spans F.) Sudden infant death syndrome

B.) Prematurity C.) Neurologic development problems F.) Sudden infant death syndrome

Although the cause of preeclampsia is not understood, there are several factors known to increase a woman's risk. Select all the following that are risk factors for preeclampsia. A.) Underweight B.) Prepregnancy diabetes C.) First baby D.) Women between the ages of 20 to 30 E.) African-American F.) Multifetal pregnancy G.) Family history

B.) Prepregnancy diabetes C.) First baby E.) African-American F.) Multifetal pregnancy G.) Family history

When providing intrapartum care for the woman with severe preeclampsia, priority nursing care is to: A.) Maintain the ordered rate of anticonvulsant medications. B.) Promote placental blood flow and prevent maternal injury. C.) Give intravenous fluids and observe urine output. D.) Reduce maternal blood pressure to prepregnancy level.

B.) Promote placental blood flow and prevent maternal injury.

A priority nursing diagnosis for a laboring woman who recently used cocaine would be: A.) Deficient knowledge. B.) Risk for injury C.) Impaired verbal communication. D.) Risk for impaired urinary elimination.

B.) Risk for injury The laboring woman who recently used cocaine is at risk for hypertensive crisis and must be protected from injury in case of seizures. She may have deficient knowledge; however, during labor is not the appropriate time to address this issue. She may have impaired verbal communication, but safety issues are a higher priority.

A newly pregnant woman calls her neighbor and volunteers to take care of her infant son while the mother cooks supper. This action is called: A.) Mimicry. B.) Role play. C.) Fantasy. D.) Search for a role fit.

B.) Role play. Role play consists of acting out some aspect of what mothers actually do. The pregnant woman searches for opportunities to hold or care for infants to practice mothering.

Choose the primary distinction between threatened and inevitable abortion. A.) Presence of cramping B.) Rupture of membranes C.) Vaginal bleeding D.) Pelvic pressure

B.) Rupture of membranes

The midwife has just examined a labor client and states that she is 10 cm dilated. The nurse is aware that this client is in which stage of labor? A.) First B.) Second C.) Third D.) Fourth

B.) Second The second stage begins with complete dilation (10 cm) and ends with the birth of the baby.

The client is unable to become pregnant after she has had one full-term pregnancy. The nurse should develop a plan of care for which health problem? A.) Primary infertility B.) Secondary infertility C.) Unexplained infertility D.) Combined factor infertility

B.) Secondary infertility

A woman with severe preeclampsia is being treated with an IV infusion of magnesium sulfate. This treatment will be evaluated as successful if: A.) Blood pressure is reduced to prepregnant baseline. B.) Seizures do not occur. C.) Deep tendon reflexes become hypotonic. D.) Diuresis reduces fluid retention.

B.) Seizures do not occur. Magnesium sulfate is a CNS depressant given primarily to prevent seizures. A temporary decrease in blood pressure can occur, but is not the purpose for giving the medication. Hypotonia is a sign of an excessive serum level of magnesium, as is depression of respiratory activity. If is critical that calcium gluconate be on hand to counteract the depressant effects of magnesium toxicity. Diuresis is not a priority outcome of magnesium sulfate administration.

What would the nurse include when teaching a pregnant client about normal changes in the cardiovascular system during pregnancy? A.) Her pulse rate will decrease. B.) She may experience dizziness if she lays on her back. C.) She will have a decrease in red blood cells. D.) She may experience a feeling of fullness in her chest.

B.) She may experience dizziness if she lays on her back.

A woman has just been admitted to the maternity unit with a diagnosis of incomplete abortion. The physician has written the following orders: (1) NPO (2) Type and crossmatch for two units of blood. (3) Start intravenous line and run Ringer's lactate at 150 mL/hr. (4) Administer Pitocin, 10 units intramuscular. (5) Acetaminophen and codeine (Tylenol with Codeine #3), every 3 to 4 hours as needed for pain (6) Bed rest with bathroom privileges Which order should the nurse carry out first for this patient? A.) Inform her of the NPO and bed rest order. B.) Start the IV and draw blood to send for the type and crossmatch. C.) Administer the pain medication. D.) Take time to listen to the client about her feelings concerning the abortion.

B.) Start the IV and draw blood to send for the type and crossmatch. Initial treatment of an incomplete abortion should focus on stabilizing the woman cardiovascularly. She may have lost blood or is at high risk for blood loss, so it is important to have her typed and crossmatched for replacement blood. The IV will help with fluid replacement.

Late deceleration patterns are noted when assessing the monitor tracing of a woman whose labor is being induced with an infusion of oxytocin (Pitocin). The woman is in a side-lying position and her vital signs are stable, falling within a normal range. Contractions are intense, last 90 seconds, and occur every 1½ to 2 minutes. The nurse's immediate action would be to: A.) Change the woman's position. B.) Stop the oxytocin. C.) Elevate the woman's legs. D.) Administer oxygen via a tight mask at 8 to 10 L/min.

B.) Stop the oxytocin. The late deceleration patterns noted are most likely related to alteration in uteroplacental perfusion associated with the strong contractions described. The immediate action would be to stop the oxytocin infusion because oxytocin stimulates the uterus to contract. The woman is already in an appropriate position for uteroplacental perfusion. Elevating her legs would be appropriate if hypotension were present. Oxygen is appropriate but not the immediate action.

A 32-week-pregnant woman calls the prenatal clinic complaining of bleeding without pain or contractions. The nurse should: A.) Tell her to rest for a couple of hours and call back if it does not stop. B.) Tell her to go to the hospital to be evaluated. C.) Make her an appointment for the next morning. D.) Have her assess fetal movement for 30 minutes.

B.) Tell her to go to the hospital to be evaluated. Signs of placenta previa are painless bleeding after 20 weeks of gestation. Active bleeding can occur; therefore she needs to be evaluated.

A client who is 6 months pregnant complains to the nurse that her husband has been working overtime lately and has applied for a second, part-time job. The nurse can best explain this action by explaining: A.) The husband is having trouble dealing with the wife's physical and emotional changes and is working more. B.) The husband is aware of the extra expenses and is trying to accept his financial responsibility as a father. C.) The husband is denying the pregnancy. D.) The husband is making a statement that he does not approve of the pregnancy.

B.) The husband is aware of the extra expenses and is trying to accept his financial responsibility as a father. Financial concerns of a pregnancy increase anxiety in some fathers. They deal with that anxiety by increasing their workload to meet the increased financial needs.

A woman and her female partner have come to the clinic stating that they would like to have a baby. During the interview, the nurse discusses the possibilities with the couple. One possibility in which the woman may conceive and deliver her biologic child is: A.) Egg donation. B.) Therapeutic insemination. C.) Surrogate parenting. D.) Ovulation induction.

B.) Therapeutic insemination. Therapeutic insemination may use semen of a donor. This can be used if a woman wants a biologic child without having a relationship with a male partner.

A client's maternal serum alpha-fetoprotein (MSAFP) levels are elevated. The nurse can anticipate that the next test done will be: A.) Amniocentesis. B.) Ultrasound. C.) Biophysical profile. D.) Chorionic villus sampling.

B.) Ultrasound. The MSAFP is a screening test, not a diagnostic test. If the levels are elevated, an ultrasound is offered to determine whether the abnormal concentration results from inaccurate gestational age, multifetal gestation, or fetal demise.

A school nurse is teaching a group of high school seniors about gynecologic care. It is important to include instructions on: A.) Scheduling a mammogram within the next 3 years. B.) Vulvar self-examination. C.) Use of contraceptives by the menopausal woman. D.) Breast self-examination.

B.) Vulvar self-examination. Vulvar self-examinations should begin in women 18 years old and in women younger than 18 if they are sexually active. Mammograms are routinely started at the age of 40. Breast self-examinations are started by women who are 20 years old. Adolescents will not benefit from information about contraceptive use by menopausal women.

A client has pregnancy-induced hypertension and is considered pre-eclamptic. The nurse has discussed dietary modifications and wishes to evaluate teaching. Which statement by the client indicates teaching was effective? A.) "I need to decrease protein to 10 percent of my diet and increase carbohydrates by 20 percent." B.) "I must restrict fluids to one liter per day and limit sodium to three grams." C.) "I should increase protein to 1.5 grams per kilogram per day and limit sodium intake to 6 grams per day or less." D,( "I need to restrict fluids to one liter per day and increase protein to 1.5 grams per kilogram per day."

C.) "I should increase protein to 1.5 grams per kilogram per day and limit sodium intake to 6 grams per day or less." Rationale: Protein increases are recommended because of losses in urine, and sodium intake is reduced slightly because of potential edema formation.

The nurse notices a paring knife lying under a laboring client's bed. The nurse attempts to remove the knife when the client's mother asks if it could remain under the bed so it will "cut the labor pains." To be sensitive to the cultural needs of the client, the nurse's best response would be: A.) "It is dangerous, so I will take it into the diet kitchen." B.) "I will put it on this shelf so it will be out of the way." C.) "Let me slide it farther under the bed, out of the way." D.) "There is no need for the knife to be there; it won't work to cut the labor pains."

C.) "Let me slide it farther under the bed, out of the way." Cultural differences need to be respected and observed if possible. The knife may offer comfort to the laboring woman. The nurse should be observant if young children are in the room to maintain safety.

After a year of infertility treatment, a woman has just begun another menstrual period. She tells the nurse, "I am so tired of trying. It feels like I will never have a baby. I just am not a real woman." The best response by the nurse would be: A.) "Remember, not everyone receiving treatment will become pregnant. We told you that at the very beginning of the treatments." B.) "Don't give up yet. You are still young and we are learning newer techniques to try every day." C.) "This must be very frustrating to you." D.) "Having a baby does not make you a woman."

C.) "This must be very frustrating to you." Infertility treatment is stressful. The woman may experience depression and guilt. The best response by the nurse is to allow the woman time to express her concerns and feelings.

On performing Leopold's maneuver on a multiparous client in early labor, the nurse finds no fetal parts in the fundus or above the symphysis. The fetal head is palpated in the right mid quadrant. The nurse notifies the admitting physician and anticipates which of the following? A.) An external version B.) An internal version C.) A cesarean delivery D.) Prolonged labor

C.) A cesarean delivery Rationale: Findings on palpation are consistent with shoulder presentation or transverse lie. Vaginal delivery is not possible, so the nurse should anticipate cesarean section.

During contractions the fetus has mechanisms in place to protect it from the decrease in blood flow. Those mechanisms include: A.) Fetal hemoglobin levels that are more resistant to oxygen. B.) Lower hemoglobin and hematocrit levels. C.) A high cardiac output level. D.) A higher respiratory level.

C.) A high cardiac output level. To prepare for labor, the fetus develops hemoglobin levels that readily take on oxygen and release carbon dioxide. The fetal hemoglobin and hematocrit levels are higher to have more oxygen-carrying capacity. The fetus has a higher cardiac output level. The fetus does not breathe yet, so there is no respiratory count.

A fetal pulse oximetry is applied to a fetus. The reading shows an oxygen saturation of 45%. The nurse realizes that this means: A.) A low reading; normal should be above 95%. B.) A high reading; normal should be between 20% and 30%. C.) A normal reading. D.) This is not conclusive.

C.) A normal reading. A normal fetal pulse oximetry reading is 30% to 70%. A normal reading for an adult is between 95% and 100%. The lower reading is considered normal because of the high hemoglobin and hematocrit levels in the fetus compared with those of an adult.

The client reports a prepregnant weight of 209 pounds and a BMI of 28. She is in her ninth month of pregnancy and has gained 30 pounds. The nurse is aware that compared with the recommended weight gain during pregnancy, this client is: A.) Within normal limits. B.) Below normal limits. C.) Above normal limits.

C.) Above normal limits. A woman with a prepregnant BMI greater than 26 should have a total recommended weight gain of 15 to 25 pounds (7 to 11.5 kg). This client has gained 30 pounds, which is above the normal recommended limits.

A woman has admitted to repeated cocaine use during her pregnancy. The nurse should be aware of which complication during labor of this mother? A.) Precipitous labor B.) Placenta previa C.) Abruptio placentae D.) Hemorrhaging

C.) Abruptio placentae Cocaine use will induce vasoconstriction of the placental vessels. This can put her at high risk for abruptio placentae.

When counseling a teenage pregnant woman, the nurse asks, "Why is eating enough protein important for the baby?" This is an example of determining which type of cognitive development? A.) Egocentrism B.) Present-future orientation C.) Abstract thinking

C.) Abstract thinking Abstract thinking is identifying cause and effect. Egocentrism involves the ability to defer personal satisfaction to respond to the needs of the infant. Present-future orientation involves the ability to make long-term plans.

The nurse noted a trace of glucose in the urine of an 8-month-pregnant woman. The nurse understands that this may be caused by: A.) Excessive sugar intake over the past few hours. B.) A sign of diabetes mellitus. The client needs further lab work. C.) An increase in the glomerular filtration rate that is normal during pregnancy. D.) An increase in the bacteria count in the urine.

C.) An increase in the glomerular filtration rate that is normal during pregnancy. An increase in the glomerular filtration rate causes the filtered glucose level to exceed the ability of the renal tubules to reabsorb it. This causes a spilling of glucose into the urine.

The best way for the nurse to evaluate the quality of a pregnant adolescent's diet is to: A.) Ask her how well she eats in a nonthreatening manner. B.) Assume it is inadequate and give her advice. C.) Ask her to describe what she ate the previous day. D.) Have her record everything she eats for 1 week.

C.) Ask her to describe what she ate the previous day. Having her recall her eating habits for 1 day will give the nurse a better understanding of her nutritional status. Not all teenagers eat unhealthy diets. Asking her how well she eats will not get an appropriate answer because teenagers are sometimes defensive in their responses and will not answer completely. The nurse will need to ask for specific information. Compliance will be low when asking her to record everything she eats for 1 week.

A pregnant client walks into the birthing center complaining of contractions. After getting her to bed, the first thing the nurse should do is: A.) Assess the mother's pulse and respirations. B.) Gather information about her medical history. C.) Assess the fetal heart rate. D.) Start an intravenous line.

C.) Assess the fetal heart rate. Assessment priorities on admission of a labor client are to determine the condition of the mother and fetus and whether birth is imminent. Checking the fetal heart rate is one of the first assessments that should be carried out. Along with assessing the fetus, the nurse should also check the maternal blood pressure and temperature.

The nurse locates fetal heart tones in the right upper quadrant of the abdomen. This finding should cause suspicion that the fetus in what presentation? A.) Occiput posterior B.) Occiput transverse C.) Breech D.) Shoulder

C.) Breech

A woman is admitted in early labor. The prenatal record states that the fetus is in a transverse lie with a shoulder presentation. The nurse can anticipate a: A.) Frequent change of positions for the mother to alter the fetal position. B.) Need for early fetal monitoring to assess for fetal heart changes. C.) Cesarean birth. D.) Prolonged second stage of labor.

C.) Cesarean birth. A transverse lie with a shoulder presentation almost always ends with a cesarean birth.

During a vaginal exam, the practitioner noted a bluish discoloration of the vagina and labia. This is considered an early sign of pregnancy and is termed: A.) Goodell's sign. B.) Hegar's sign. C.) Chadwick's sign.

C.) Chadwick's sign. Chadwick's sign is a bluish purple color of the vagina and labia caused by the congestion of blood in the area. Goodell's sign is cervical softening. Hegar's sign is the softening of the lower uterine segment due to pregnancy.

A client is being discharged, having been diagnosed with false labor. The nursing diagnosis for her is Deficient Knowledge: characteristics of true labor. An appropriate expected outcome for this diagnosis is that the: A.) Client will return to the hospital when she is in true labor. B.) Client will define true labor. C.) Client will describe reasons for returning to the hospital for evaluation. D.) Client will be able to determine false from true labor.

C.) Client will describe reasons for returning to the hospital for evaluation. The client may not be able to determine true from false labor; however, she should be made aware of what signs to look for that may indicate the need for evaluation.

The midwife records that the client's cervix is "100%, 5 cm." The nurse understands that the client's cervix is: A.) Completely dilated and effaced. B.) Completely dilated and half-effaced. C.) Completely effaced and half-dilated. D.) Half-dilated and half-effaced.

C.) Completely effaced and half-dilated. Effacement is measured in percentages. The fully thinned cervix is 100% effaced. The dilation is measured in centimeters; dilation goes from closed to 10 cm. This client is completely effaced and halfway dilated.

A new antepartal client states that she is about 15 weeks pregnant. The nurse palpates the fundus midway between the symphysis pubis and umbilicus. The nurse best interprets these findings as: A.) Not congruent; the fundal height is too high for the age of the pregnancy. B.) Not congruent; the fundal height is too low for the age of the pregnancy. C.) Congruent; the fundal height is at the level appropriate for the age of the pregnancy.

C.) Congruent; the fundal height is at the level appropriate for the age of the pregnancy. The fundal height should be midway between the symphysis pubis and umbilicus at about 16 weeks.

The onset of late decelerations on the fetal monitor should lead the nurse to suspect which condition? A.) Head compression B.) Cord compression C.) Decreased uteroplacental blood flow D.) Close uterine contractions

C.) Decreased uteroplacental blood flow

The nurse formulates the nursing diagnosis Risk for Impaired Parenting related to knowledge deficit in newborn care for a 15-year-old primipara who delivered yesterday. Which is the most appropriate intervention when planning this client's discharge teaching? A.) Have the client watch a video on newborn care. B.) Give her information about a support group for adolescent mothers. C.) Demonstrate how to care for the newborn and have the client return the demonstration. D.) Give the client printed instructions on newborn care.

C.) Demonstrate how to care for the newborn and have the client return the demonstration.

During the third trimester, it is normal for a woman to feel: A.) Depressed because of a change in the body image. B.) Introverted and concerned about herself. C.) Dependent on her partner and family. D.) Ambivalent about the pregnancy.

C.) Dependent on her partner and family. During the third trimester, the expectant mother often becomes increasingly dependent on her partner.

A pregnant client has a 2-year history of uncontrolled hypertension. The nurse can anticipate that which fetal study to be ordered? A.) Amniocentesis B.) Chorionic villus sampling C.) Doppler ultrasound blood flow assessment

C.) Doppler ultrasound blood flow assessment Pregnancies complicated by hypertension may have a Doppler ultrasound assessment of blood flow through the umbilical artery done to identify abnormalities in the diastolic flow.

When doing a chart review of a client with preeclampsia, the nurse noted that the client was assessed with +3 edema. This indicates: A.) Minimal edema of the lower extremities. B.) Marked edema of lower extremities. C.) Edema of lower extremities, face, hands, and sacral area. D.) Generalized massive edema that includes ascites.

C.) Edema of lower extremities, face, hands, and sacral area. The +3 assessment of edema indicates edema of the lower extremities, face, hands, and sacral area. Option A indicates A +1 grade, option B indicates a +2 grade, and option D indicates a +4 grade.

The lab work for a new antepartal client shows a hemoglobin level of 12 g/dL. The client is concerned that she is anemic. The nurse should base the answer on which of the following? A.) A hemoglobin level of 12 g/dL is below normal and may be a sign of iron deficiency anemia. B.) A hemoglobin level of 12 g/dL is above the normal level and is reassuring. C.) Even though a hemoglobin level of 12 g/dL is low for an adult, it is within normal limits for the first trimester of a pregnancy. D.) Even though a hemoglobin level of 12 g/dL is high for an adult, it is within normal limits for the first trimester of a pregnancy.

C.) Even though a hemoglobin level of 12 g/dL is low for an adult, it is within normal limits for the first trimester of a pregnancy. During the first trimester of pregnancy, a hemoglobin level below 11 g/dL is necessary for a diagnosis of iron deficiency anemia. The normal hemoglobin level is lower during pregnancy because of the greatly expanded plasma volume.

The antepartal client complains of burning and numbness in her left hand. She is diagnosed with carpal tunnel syndrome. The nurse can explain this is because of: A.) Excessive cortisol production during pregnancy. B.) Decrease in central nervous system stimulation during pregnancy. C.) Excessive edema during pregnancy, compressing the median nerve. D.) Production of relaxin during pregnancy.

C.) Excessive edema during pregnancy, compressing the median nerve. Fluid retention during pregnancy is associated with carpal tunnel syndrome. The edema compresses the median nerve.

During pregnancy, if a woman restricts her fat intake, she may develop an essential fatty acid deficit. One essential fatty acid, docosahexaenoic acid (DHA), is important during pregnancy for the formation of the: A.) Fetal liver. B.) Maternal red blood cells. C.) Fetal brain. D.) Maternal uterine lining.

C.) Fetal brain. DHA is important in the development of the placenta, fetal visual function, and brain development. Women should be encouraged to include fish and red meat in their diets several times per week to maintain the proper level of DHA.

An amniocentesis is ordered for an 8-month-pregnant client. The nurse is aware that the most common reason for this test at this time is to test for: A.) Fetal demise. B.) Fetal chromosome abnormalities. C.) Fetal lung maturity. D.) Amniotic fluid amount.

C.) Fetal lung maturity. A common indication for amniocentesis during the third trimester is to determine fetal lung maturity. Amniocentesis may be done midtrimester to determine chromosomal abnormalities. Amniocentesis is not done to determine fetal demise or amniotic fluid amount.

The nurse is teaching a group of expectant parents about cardinal movements or changes in position, that occur as the fetus with a cephalic presentation passes through the birth canal. Order the cardinal movements in proper sequence for the nurse's presentation. All options must be used. A.) Expulsion B.) External rotation C.) Flexion D.) Internal rotation E.) Restitution

C.) Flexion D.) Internal rotation E.) Restitution B.) External rotation A.) Expulsion

A woman is born without a functioning uterus but has functioning ovaries. She and her husband come to the infertility clinic requesting information on the possibility of having a baby. One possibility for this couple to have a biological child is: A.) In vitro fertilization. B.) Therapeutic insemination. C.) Gestational surrogate. D.) Gamete intrafallopian transfer.

C.) Gestational surrogate. A surrogate mother in a gestational surrogate pregnancy supplies her uterus only, with the infertile couple supplying the sperm and ovum. The other choices all require a functioning uterus in the biologic mother.

A pregnant client has undergone a glucose tolerance test. The results are 105 milligrams per deciliter-fasting, 200-1 hour, 175-2 hours, and 160-3 hours. How would the nurse interpret the meaning of these results? A.) Glucose levels are within normal limits; therefore, the client does not have gestational diabetes mellitus (GDM). B.) Glucose levels indicate that the client is hypoglycemic and dietary alterations should be instituted. C.) Glucose levels indicate GDM and treatment should be initiated. D.) Glucose levels are inconclusive and the test needs to be repeated.

C.) Glucose levels indicate GDM and treatment should be initiated. Rationale: Values that meet or exceed two or more of the designated glucose parameters are considered diagnostic for GDM. This client exceeds parameters in all four levels (i.e., greater than or equal to 95, greater than or equal to 80, greater than or equal to 155, greater than or equal to 140).

All the following women in labor are requesting pain medication. To which one should the nurse administer an opioid analgesic first? A.) Primigravida, 2 cm dilated, 50% effaced, grimacing slightly with each contraction B.) Gravida 4, 9 cm dilated, 100% effaced, wants to push with each contraction C.) Gravida 2, 6 cm dilated, 100% effaced, rocks back and forth in bed with each contraction D.) Primigravida, 1 cm dilated, moans loudly with each contraction, has present history of heroin use

C.) Gravida 2, 6 cm dilated, 100% effaced, rocks back and forth in bed with each contraction The gravida 2 is well established into the labor and the medication will not slow the contractions. The primigravida who is 2 cm dilated is too early into the labor; the medication may slow or stop her contractions. The gravida 4 is too near birth and the medication may affect the newborn's respiratory effort. The primigravida who is 1 cm dilated has a history of heroin use; further opioid medication is not recommended.

The pregnant client reports that she has a 3-year-old child at home who was born at term, had a miscarriage at 10 weeks' gestation, and delivered a set of twins at 28 weeks' gestation who died within 24 hours. On the prenatal health record, the nurse would record this information using which entry? A.) Gravida 2, para 1 B.) Gravida 3, para 3 C.) Gravida 4, para 2 D.) Gravida 5, para 4

C.) Gravida 4, para 2 Rationale: Counting the current pregnancy, the client has been pregnant a total of four times for gravida 4. Para is the number of pregnancies that have reached viability, in this case two.

A client, 9 months pregnant, calls the clinic with complaints of urinary frequency, urgency, and pain. The nurse's next action should be to: A.) Reassure the client that this is normal after lightening occurs during the last month of pregnancy. B.) Refer the client to a specialist because this is a sign of kidney failure. C.) Have the client come in to be assessed for a urinary tract infection. D.) Assure the client that the problem can wait until her next regular appointment.

C.) Have the client come in to be assessed for a urinary tract infection. Frequency and urgency are normal after lightening because of pressure against the bladder. However, the complaint of pain may indicate a urinary tract infection.

During the active stage of labor the woman is using a rapid "pant-blow" breathing pattern. She starts to complain of feeling dizzy and has some numbness in her fingers. The nurse's next action should be to: A.) Notify the physician. B.) Do a vaginal exam to check for the progression of labor. C.) Have the woman breathe into a paper bag. D.) Offer pain medication.

C.) Have the woman breathe into a paper bag. Hyperventilation is common when breathing techniques are used. It results from rapid deep breathing that causes excessive loss of carbon dioxide and therefore respiratory alkalosis. Blowing into a paper bag or her own cupped hands will increase her carbon dioxide level by having the woman rebreathe her exhaled air.

A primigravida is admitted in early labor. The nurse notices on the prenatal record that the position of the fetus is left occiput posterior. Because of this information, the nurse can anticipate: A.) A cesarean section. B.) A short labor and birth process. C.) Increased back pain with labor. D.) A short labor with a prolonged birth process.

C.) Increased back pain with labor. When the fetus is in the posterior position, the labor may be longer and more uncomfortable. Back discomfort increases with contractions and will continue between contractions. The fetus may not be able to deliver until it rotates into the anterior position.

A client comes to the clinic requesting an ultrasound to confirm a pregnancy. Her last menstrual period was 15 days ago. The nurse's best response would be to: A.) Make an appointment. B.) Ask the reason for needing a confirmation this soon in the pregnancy. C.) Inform her that an ultrasound cannot confirm a pregnancy this early. D.) Refer her to a specialist who is trained in confirming early pregnancies.

C.) Inform her that an ultrasound cannot confirm a pregnancy this early. The embryo is not visualized until 5 to 6 weeks after the last menstrual period.

A primigravida is in the latent phase of labor and is at low risk for complications of labor. She asks the nurse if she may walk for a few minutes. The nurse is aware that this is (is not) possible because: A.) Continuous fetal monitoring is required. B.) Continuous monitoring of the contractions is necessary at this stage of labor. C.) Intermittent auscultation of fetal heart rate is appropriate for her. D.) There is no need to assess fetal heart rate at this early stage of labor.

C.) Intermittent auscultation of fetal heart rate is appropriate for her. Continuous fetal and uterine monitoring are not necessary for the latent phase of labor in women who are at low risk for complications.

The client is in active labor and the nurse is receiving an order for an opioid analgesic from the health care provider. The nurse verifies that the order is for which priority route for administration? A.) Intramuscular (IM) B.) Oral (PO) C.) Intravenous (IV) D.) Rectal (PR)

C.) Intravenous (IV) Rationale: Opioid analgesics given for pain relief during labor are most often given by the IV route so that the medication will have a rapid onset and a relatively short half-life. This desired drug profile will provide maternal benefit while preventing neonatal respiratory depression.

A client, 6 months pregnant, is concerned because a mole on her abdomen is becoming darker. She has heard that this is a sign of cancer. The nurse should base the answer on which of the following? A.) This is a sign of cancer and the mole should be evaluated by a specialist. B.) It is common for moles to become darker during pregnancy because of the increase in hormones; thus, no action is necessary. C.) It is common for moles to become darker during pregnancy; however, it should be evaluated by the physician. D.) It is a sign of integumentary changes that need to be assessed within the next 2 weeks.

C.) It is common for moles to become darker during pregnancy; however, it should be evaluated by the physician. The increased levels of estrogen, progesterone, and melanocyte-stimulating hormone increase pigmentation in 90% of the pregnant woman. Preexisting moles become darker during pregnancy. However, there is a small risk that a cancer is forming and the client needs to have the mole assessed.

When doing a vaginal exam, the nurse notes a triangular-shaped depression toward the mother's left side and pointing up toward her abdomen. The nurse can record the fetal position as: A.) LOP. B.) ROP. C.) LOA. D.) ROP.

C.) LOA. The triangular shape is the posterior fontanel, which makes the positioning part the occiput. The posterior fontanel is toward the mother's left side and anterior. This makes the position left occiput anterior (LOA).

Firm contractions that occur every 3 minutes and last 100 seconds may reduce fetal oxygen supply because they: A.) Cause fetal bradycardia and reduce oxygen concentration. B.) Activate the fetal sympathetic nervous system. C.) Limit the time for oxygen exchange in the placenta. D.) Suppress the normal variability of the fetal heart.

C.) Limit the time for oxygen exchange in the placenta. The resting time between these contractions is about 80 seconds, which reduces the time available for exchange of oxygen and waste products in the placenta. This will reduce the fetal oxygen supply. The other choices can all be results of the decreased oxygen supply.

A woman who is about 37 weeks' gestation tells the nurse that for some reason this morning she can breathe easier. The nurse can best explain this as being a: A.) Concern, and the fetus needs to be assessed. B.) Normal change toward the end of the pregnancy caused by a decreased use of oxygen by the fetus. C.) Normal change because of the fetus's dropping down into the pelvis region, relieving the pressure on her diaphragm. D.) Normal change caused by the maternal cardiac output increasing as she gets closer to labor.

C.) Normal change because of the fetus's dropping down into the pelvis region, relieving the pressure on her diaphragm. Lightening occurs toward the end of the pregnancy as the fetus descends toward the pelvic inlet. When this occurs, the woman notices that she breathes more easily because upward pressure on her diaphragm is reduced.

A 25-year-old woman is in for her first gynecologic examination. She is in good health and has no family history of cancer or reproductive diseases. She asks the nurse if a mammogram will be performed. The nurse is aware that this woman will: A.) Need a mammogram this year, and it should be repeated every 2 years. B.) Need a mammogram this year, and it should be repeated every 5 years. C.) Not need a mammogram until she is 40 years old. D.) Not need a mammogram until she is 30 years old.

C.) Not need a mammogram until she is 40 years old. The American Cancer Society and American College of Obstetricians and Gynecologists recommends that mammograms start at the age of 40 unless the woman is at high risk for breast cancer.

During labor, a woman suddenly complains of increasing pain, and the electronic monitor shows no uterine activity. The abdomen is boardlike and tender and the fetal heart tones show late decelerations. The nurse should: A.) Turn the mother to her side and continue to monitor the fetal heart rate. B.) Assess the mother's blood pressure, temperature, pulse, and respirations. C.) Notify the health care provider. D.) Anticipate that the woman has moved into the second stage of labor.

C.) Notify the health care provider. With abruptio placentae, the uterus may become exceedingly firm and tender. Because of decreased blood flow, the fetus will show signs of hypoxia. An immediate cesarean birth may be necessary; therefore the health care provider should be notified.

A postmenopausal client is just learning to do breast self-examination (BSE). To aid in remembering to perform the procedure, at which time should the nurse recommend that the client perform BSE? A.) Weekly just before grocery shopping B.) On a random day once each month according to convenience C.) Once a month on a standard day that the client can remember D.) Just prior to each 6-month check-up for another identified health problems

C.) Once a month on a standard day that the client can remember

After birth, the woman complains of chills. The first intervention by the nurse should be to: A.) Monitor the maternal temperature. B.) Monitor the maternal blood pressure. C.) Place a warm blanket on the woman. D.) Explain to the woman this is caused by the excitement of birth and will stop in about 30 minutes.

C.) Place a warm blanket on the woman. Many women are chilled after birth. The cause of this reaction is unknown but probably relates to the sudden decrease in effort, loss of the heat produced by the fetus, decrease in intraabdominal pressure, and fetal blood cells entering the maternal circulation. The chill lasts for about 20 minutes and subsides spontaneously. A warm blanket may help shorten the chill.

In reviewing the chart of a prenatal client, which client finding would be considered by the nurse to be a probable sign of pregnancy? A.) Fetal heartbeat on ultrasound B.) Amenorrhea C.) Positive pregnancy test D.) Chloasma

C.) Positive pregnancy test Rationale: Probable signs of pregnancy are those that are detected by the examiner and are usually related to the physical signs of pregnancy.

Signs of a threatened abortion are noted in a woman at 8 weeks of gestation. Which of the following is an appropriate management approach for this type of abortion? A.) Prepare the woman for a dilation and curettage (D&C). B.) Place the woman on bed rest for at least 1 week and reevaluate. C.) Prepare the woman for an ultrasound to determine the integrity of the gestational sac. D.) Comfort the woman by telling her that if she loses this baby, she can try to get pregnant again in about 1 month.

C.) Prepare the woman for an ultrasound to determine the integrity of the gestational sac. A D&C is not considered until signs of progress to inevitable abortion are noted or the contents expelled are incomplete. Bed rest is not recommended for this woman, just a decrease in activities. Telling the woman she can get pregnant again soon is not a therapeutic response because it discounts the importance of this pregnancy. If the pregnancy is lost, she should be guided through the grieving process.

A woman must have general anesthesia for a planned cesarean birth because of a previous back surgery. The nurse should therefore expect to administer: A.) Naltrexone (Trexan). B.) An oral barbiturate. C.) Ranitidine (Zantac). D.) Promethazine (Phenergan).

C.) Ranitidine (Zantac). During general anesthesia, there is a risk for maternal aspiration. To prevent lung injury if aspiration occurs, drugs may be given to raise the gastric pH and make secretions less acidic, such as ranitidine. Naltrexone is an opioid antagonist, promethazine is used to relieve nausea, and barbiturates are sedating.

Choose the correct client teaching to follow amniocentesis. A.) Drink 1 to 2 quarts of clear fluid to replace fluid taken in the procedure. B.) Resume all normal activities when desired. C.) Report persistent contractions, vaginal bleeding, fluid leaking, or fever. D.) Eat a diet with increased iron for the 2 days after amniocentesis.

C.) Report persistent contractions, vaginal bleeding, fluid leaking, or fever.

A client is receiving magnesium sulfate for severe preeclampsia. What nursing actions are appropriate interventions? Select all that apply. A.) Limit fluid intake to 1000 mL/24 hours. B.) Prepare for the possibility of a precipitate delivery. C.) Restrict visitors and keep the room darkened and quiet. D.) Obtain calcium gluconate for use as an antagonist if necessary. E.) Assess for patellar reflexes.

C.) Restrict visitors and keep the room darkened and quiet. D.) Obtain calcium gluconate for use as an antagonist if necessary. E.) Assess for patellar reflexes.

You are observing for fetal heart rate (FHR) accelerations in a nonstress test (NST) for a woman who is 26 weeks pregnant. The average FHR baseline is 145 to 155 beats per minute (bpm). Within 20 minutes, the FHR accelerated to 165 bpm six times, for 10 to 15 seconds. How should you interpret this information? A.) Results are nonreassuring, and another 20 minutes of monitoring is needed. B.) Results are nonreassuring because of too few accelerations within the time period. C.) Results are reassuring because the FHR accelerated by 10 bpm for 10 seconds. D.) Results are reassuring because the fetus was inactive during the monitoring.

C.) Results are reassuring because the FHR accelerated by 10 bpm for 10 seconds.

A pregnant woman at 14 weeks of gestation is admitted to the hospital with a diagnosis of hyperemesis gravidarum. The primary goal of her treatment at this time would be to: A.) Rest the gastrointestinal tract by restricting all oral intake for 48 hours. B.) Reduce emotional stress by encouraging the woman to discuss her feelings. C.) Reverse fluid, electrolyte, and acid-base imbalances that are present. D.) Restore the woman's ability to take and retain oral fluid and foods.

C.) Reverse fluid, electrolyte, and acid-base imbalances that are present. Fluid, electrolyte, and acid-base imbalances present the greatest immediate danger to the well-being of the maternal-fetal unit. Options A, B, and D are all components of treatment but do not represent immediate care for patients with hyperemesis gravidarum.

A school nurse has finished conducting a teaching session with high school girls about breast self-examination (BSE). The nurse concludes that the information was learned correctly when one of the girls states to do the exam at which time? A.) Once per month when the client thinks she is ovulating B.) On the first day of each month C.) Seven days after menstruation begins D.) On the first day of the menstrual cycle

C.) Seven days after menstruation begins

A woman has just had a spontaneous abortion. She asks the nurse, "Why did this happen?" The nurse is aware that the most common cause of spontaneous abortion is: A.) Improper maternal nutrition. B.) Caffeine use in the early pregnancy. C.) Severe congenital abnormalities. D.) Improper implantation.

C.) Severe congenital abnormalities. Chromosomal abnormalities account for about 50% to 60% of early spontaneous abortions. Other possible causes are various types of infections and maternal disorders.

A client, 5 months pregnant, confides to the nurse that she drinks one glass of wine every night with dinner. The best recommendation by the nurse should be: A.) One glass of wine per day is within the acceptable limits. B.) The client should drink only one glass of wine every other day. C.) The client should not be drinking any alcohol during the pregnancy. D.) Alcohol only damages the fetus during the first trimester, so because she is in her 5th month, one glass of wine will not harm the fetus.

C.) The client should not be drinking any alcohol during the pregnancy. There is an association between drinking and fetal alcohol syndrome; alcohol should be completely avoided during pregnancy. Alcohol also interferes with the absorption and use of protein, thiamine, folic acid, and zinc.

A pregnant woman may assume a wide stance when standing and have a waddling gait when walking. This is caused by: A.) Loss of calcium in the bones when it is transferred to the fetus. B.) Strengthening of the connective tissue around the pelvis. C.) The effort to compensate for a changing center of gravity. D.) The strain on the muscles and ligaments of the back.

C.) The effort to compensate for a changing center of gravity. The change of the center of gravity and muscle fatigue cause a wide stance and waddling gait during pregnancy.

The laboring client is 8 centimeters dilated, 100% effaced, with vertex presenting at +2 station. The fetal heart rate gradually slows during each contraction, returning to baseline by the end of the contraction. The nurse concludes that which of the following is occurring? A.) The umbilical cord is becoming compressed. B.) There is uteroplacental insufficiency. C.) The fetal head is becoming compressed. D.) The fetus is moving between contractions.

C.) The fetal head is becoming compressed.

The nurse is examining a client who is at 12 weeks' gestation. The examiner would expect to find the fundus at which location at this time? A.) 3 cm below the sternum B.) The level of the umbilicus C.) The level of the symphysis pubis D.) 3 cm below the umbilicus

C.) The level of the symphysis pubis

In which of the following situations would fetal oxygenation be compromised? A.) The mother has been taking Tylenol for mild headaches 2 days prior to the onset of labor. B.) The mother regularly sleeps on her left side. C.) The mother routinely uses cocaine. D.) The mother drinks one caffeine drink a day.

C.) The mother routinely uses cocaine. Cocaine use produces maternal hypertension. Hypertension in the mother reduces blood flow to the placenta and decreases the fetal oxygenation.

The nurse concludes that client teaching has been effective when the laboring client's partner shouts, "She's crowning!" as what event occurs? A.) He first starts to see a little of the baby's head. B.) The baby's head recedes upward between pushing contractions. C.) The perineum is thin and stretching around the occiput. D.) The mouth and nose are being suctioned.

C.) The perineum is thin and stretching around the occiput. Rationale: Crowning is the point in time when the perineum is thin and stretching around the fetal head both between and during contractions. Delivery is imminent when crowning occurs.

After 2 years of infertility treatment, a couple decides that they cannot financially go through any more treatments. When discussing this decision with the couple, the nurse can discuss with them: A.) The possibility that accepting the fact they may not have a child will cause them to relax and perhaps conceive. B.) How to accept the fact they may never have a child. C.) The possibility of adoption. D.) The possibility of accepting a niece or nephew as a surrogate child.

C.) The possibility of adoption. Not every couple who seeks treatment for infertility achieves a "take home" baby. Adoption may become an option for these couples. The nurse can assist them to explore their personal feelings about adoption, availability of newborns compared with older children, and the pros and cons of adoption.

A mother who just gave birth to her second son may have feelings of grief because: A.) It is typical to desire a girl when there is already a son in the family. B.) Two children means less time to pursue the mother's ambitions. C.) There is a loss between herself and the first child; she thinks that the older child is growing up and away from her. D.) She is concerned there will be less time for a relationship with her husband.

C.) There is a loss between herself and the first child; she thinks that the older child is growing up and away from her. The woman giving birth to another child spends a great deal of time working out a new relationship with the first child. This behavior may foster feelings of guilt as she tries to expand her love to include the second child.

If "nesting" is an accepted cultural norm, it will most likely occur at which time during the pregnancy? A.) First trimester B.) Second trimester C.) Third trimester D.) Anytime throughout the pregnancy

C.) Third trimester Nesting is a third-trimester phenomenon. The baby's room and necessities will be finalized at this time.

The nurse is reviewing laboratory reports from several clients who had Pap tests done 3 days ago. One result stated, "high-grade squamous intraepithelial lesion." The nurse is aware that this report indicates: A.) Negative results and no follow-up is required. B.) A negative result but a 3-month repeat Pap test should be done. C.) This result has a high likelihood of becoming cancerous, and a follow-up is necessary for treatment. D.) The results are inconclusive and the woman should have a repeat test done in 6 months.

C.) This result has a high likelihood of becoming cancerous, and a follow-up is necessary for treatment. High-grade squamous intraepithelial lesion was previously categorized as carcinoma in situ. These cell changes are likely to become cancerous without definitive treatment. This woman requires immediate follow-up on the results.

A 7-month-pregnant client is concerned about how her 3-year-old will adapt to the new baby. One suggestion the nurse can give is: A.) Not to talk to the toddler about the new baby until it is born because toddlers do not understand the concept of newborns. B.) To have the toddler help pack up all the toddler's baby toys to give to the new baby. C.) To move the toddler to a "big girl" bed now so the toddler won't associate the new baby with the loss of her crib. D.) To teach the toddler that sharing will be important when the new baby arrives.

C.) To move the toddler to a "big girl" bed now so the toddler won't associate the new baby with the loss of her crib. Any changes in sleeping arrangements should be made several weeks before the birth so that the toddler does not feel displaced by the new baby. The toddler needs to be told of the new baby close to the time of birth because toddlers do not have a clear perception of time. Toddlers do not understand the concept of sharing and may have feelings of jealousy and resentment if the baby takes over the toddler's toys.

During labor, the nurse notices that the woman's support partner touches her lightly during contractions. When the woman is touched, she relaxes her muscles. The nurse realizes that the couple is using the technique of: A.) Progressive relaxation. B.) Neuromuscular disassociation. C.) Touch relaxation. D.) Relaxation against pain.

C.) Touch relaxation. During touch relaxation, the woman loosens taut muscles when they are touched by her partner. After practice during the pregnancy, the woman becomes conditioned to recognize the touch of her partner as a signal for the release of tension.

A 36-year-old primigravida is in the clinic for her first prenatal appointment. The nurse can anticipate that the multiple-marker screening may be done on this client to screen for: A.) Gestational diabetes. B.) Hypertensive disease of pregnancy. C.) Trisomy disorders. D.) Placenta previa.

C.) Trisomy disorders. The multiple-marker screening test (MSAFP, hCG, unconjugated estriol, inhibin A) is used to screen for trisomies in the fetus. Older primigravidas are at higher risk for these defects.

The client, 8 months pregnant, comes into the clinic for her regular appointment. She is lying down in the supine position on the exam table and starts to complain about nausea and dizziness. Her skin is cool and clammy. The next action by the nurse should be to: A.) Take the client's vital signs. B.) Listen to the fetal heart rate. C.) Turn the client onto her side. D.) Wipe her face off with a cool cloth.

C.) Turn the client onto her side. During the second and third trimesters, the weight of the gravid uterus can partially occlude the vena cava when the woman is lying supine. This causes a drop in maternal blood pressure, decrease in cardiac output, and decrease in blood flow to the fetus. The corrective action is to advise the pregnant woman never to lie on her back unless a wedge or pillow is placed under the right hip to displace the uterus away from the vena cava.

On admission to the labor suite, the woman begins to cry out loudly, "Lord help me, I am going to die." She repeats this phase loudly with each contraction. The nurse's best response would be to: A.) Explain to the woman that she is disturbing other clients. B.) Praise her between contractions when she is quiet. C.) Understand that this may be a cultural mannerism and accept her individual response to labor. D.) Understand that this may be a cultural mannerism and do client teaching to help her understand other ways of expressing her fear and pain.

C.) Understand that this may be a cultural mannerism and accept her individual response to labor. Women should be encouraged to express themselves in any way they find comforting. The cultural diversity of their expressions must be respected. Accepting a woman's individual response to labor and pain promotes a therapeutic relationship. Belittling her, praising her falsely, or trying to show her a "better way" of dealing with the pain will interfere with the therapeutic relationship and lower the woman's self-esteem.

Before administering IV magnesium sulfate therapy to a client with preeclampsia which parameters should the nurse consider as the highest priority? A.) Urinary glucose, acetone, and specific gravity B.) Temperature, blood pressure, and respirations C.) Urinary output, respirations, and patellar reflexes D.) Level of consciousness, funduscopic appearance, and knee reflex

C.) Urinary output, respirations, and patellar reflexes

Which of the following statements indicates the primary focus of the pregnant woman during the first trimester? A.) "How is the baby's heart today?" B.) "Is the baby growing at the right rate?" C.) "Since I am not eating much because of the morning sickness, is the baby malnourished?" D.) "Why are my breasts so tender?"

D.) "Why are my breasts so tender?" During the first trimester, the primary focus of the woman is on herself, not the fetus.

A 48-year-old woman asks the nurse how often she should see the physician for a breast exam. The nurse is aware that a woman at that age should have a clinical breast examination every: A.) 5 years. B.) 3 years. C.) 2 years. D.) 1 year.

D.) 1 year. Clinical breast examinations should be performed every 3 years for women 20 to 39 years of age and annually for woman 40 years or older.

The physician has ordered a chorionic villus sampling to be done on a client. The nurse knows to schedule the test to be done between which weeks of gestation? A.) 4 and 6 weeks B.) 6 and 8 weeks C.) 8 and 10 weeks D.) 10 and 12 weeks

D.) 10 and 12 weeks CVS is usually performed between 10 and 12 weeks of gestation. At this time the chorionic villi have been formed significantly to obtain adequate samplings.

A woman pregnant for the first time is concerned because she has not felt the baby move. She tells the nurse that her mother told her this is a bad sign and that something may be wrong with the baby. The woman is 12 weeks pregnant. The best response by the nurse should be based on the knowledge that quickening occurs at: A.) 10 to 12 weeks. B.) 12 to 14 weeks. C.) 14 to 16 weeks. D.) 16 to 20 weeks.

D.) 16 to 20 weeks. Quickening occurs from about weeks 16 to 20. The primigravida will notice the fetal movements later than a multigravida.

The fetal heart rate (FHR) shows variable decelerations. Which activity carried out by the nurse could decrease or eliminate this pattern? A.) Encourage woman to relax in a warm shower. B.) Apply a fetal scalp electrode. C.) Assist client into a supine position. D.) Begin amnio-infusion of normal saline.

D.) Begin amnio-infusion of normal saline.

A primigravida asks the nurse about signs she can look for that would indicate that the onset of labor is getting closer. The nurse should describe which of the following? A.) Weight gain of 1 to 3 pounds B.) Quickening C.) Fatigue and lethargy D.) Bloody show

D.) Bloody show Premonitory signs of labor (prodromal labor) include weight loss of 1 to 3 pounds, a burst of energy or the nesting instinct, and passage of the mucus plug (also termed pink or bloody show) as the cervix ripens. Quickening is the perception of fetal movement by the mother, which occurs at 16 to 20 weeks' gestation.

A prenatal client states she is lactose- intolerant. Which of the following foods may be suggested to her to increase her calcium intake? A.) Lean chicken B.) Salmon C.) Apple juice D.) Broccoli

D.) Broccoli 2½ cups of broccoli have the calcium equivalent of 1 cup of milk.

During vaginal examination, the nurse palpates the fetal head and a large, diamond-shaped fontanelle. The nurse documents the fetal presentation as which of the following? A.) Shoulder B.) Breech C.) Vertex D.) Brow

D.) Brow Rationale: In a brow presentation, the fetal forehead and the large, diamond-shaped anterior fontanelle are palpated during vaginal exam.

The nurse notes a pattern of decelerations on the fetal monitor that begins shortly after the contraction begins and returns to baseline just before the contraction is over. The correct nursing response is to: A.) Give the woman oxygen by face mask at 8 to 10 L/min. B.) Position the woman on her side. C.) Increase the rate of the woman's IV fluid. D.) Continue to observe and record the normal pattern.

D.) Continue to observe and record the normal pattern.

The fetal heart rate (FHR) baseline is 145 beats per minute with short-term variability of 12 beats per minute. The nurse should take which action at this time? A.) Notify client's health care provider immediately. B.) Obtain order to start IV of lactated Ringer's solution. C.) Reposition client on her hands and knees. D.) Encourage continued use of breathing and relaxation techniques.

D.) Encourage continued use of breathing and relaxation techniques.

When obtaining a reproductive health history from a female patient, the nurse should: A.) Limit the time spent on exploration of intimate topics. B.) Avoid asking questions that might embarrass the patient. C.) Use only accepted medical terminology when referring to body parts and functions. D.) Explain the purpose for the question asked and how the information will be used.

D.) Explain the purpose for the question asked and how the information will be used. Sufficient time must be spent on gathering relevant data, even if it may be embarrassing for the patient or the nurse or involves intimate topics. Always use terms the patient can understand.

When obtaining a reproductive health history from a female patient, the nurse should: A.) Limit the time spent on exploration of intimate topics B.) Avoid asking questions that may embarrass the patient. C.) Use only accepted medical terminology when referring to body parts and functions. D.) Explain the purpose for the questions asked and how the information will be used.

D.) Explain the purpose for the questions asked and how the information will be used. Sufficient time must be spent on gathering relevant data, even if it may be embarrassing for the patient or the nurse or involves intimate topics. Always use terms the patient can understand.

A couple is in for fertility counseling. They have achieved pregnancy six times, but have lost each pregnancy before the 20th week of gestation. The nurse is aware that the most common cause of repeated pregnancy loss is: A.) Lack of FSH hormone. B.) Tubal obstruction. C.) Dysfunction in the pituitary gland. D.) Fetal chromosome defects.

D.) Fetal chromosome defects. Errors in the fetal chromosomes may result in spontaneous abortion, usually in the first trimester. Lack of FSH and dysfunction of the pituitary gland are causes of disorders in ovulation; tubal obstruction prevents implantation

A station of +1 means that the: A.) Maternal cervix is open 1 cm. B.) Mother's ischial spines project into her pelvis 1 cm. C.) Fetus is unlikely to be born vaginally because the pelvis is small. D.) Fetal presenting part is 1 cm below the mother's ischial spines.

D.) Fetal presenting part is 1 cm below the mother's ischial spines.

A new client tells the nurse that she has a 7-year-old boy born full term, a 6-year-old girl was born 5 weeks premature, and 5-year-old twin boys who were 6 weeks premature, and that had a therapeutic abortion last year. In using the GTPAL method of pregnancy outcomes, this client would be: A.) G:5, T:1, P:3, A:1, L:3. B.) G:5, T:1, P:2, A:1, L:4. C.) G:4, T:1, P:2, A:1, L:3. D.) G:4, T:1, P:2, A:1, L:4.

D.) G:4, T:1, P:2, A:1, L:4. GTPAL is used to determine pregnancy outcomes, not number of babies. Gravida (G) is the number of times pregnant; term (T) is the number of pregnancies that have gone to term; preterm (P) is the number of pregnancies that were delivered prior to term; abortion (A) is the number of pregnancies that ended prior to 20 weeks' gestation; and living children (L) is the number of pregnancies that delivered living babies.

The nurse is on the planning committee to establish a prenatal clinic for teenage mothers. Which of the following suggestions would be most helpful in eliminating barriers for the teenager seeking health care in this planned clinic? A.) Place the clinic away from the school so the teenager will not have to worry about being seen by others. B.) Have the clinic located in the country so the teenager may find the environment peaceful. C.) Have the health care workers trained to teach why early sexual relations are harmful. D.) Have the clinic funded to operate in the evenings and on Saturday.

D.) Have the clinic funded to operate in the evenings and on Saturday. The two major barriers to health care for teenagers are scheduling conflicts and negative attitudes of health care workers. To help the teenager, the clinic should be located close to her, convenient to public transportation, and be open after school hours.

The nurse should realize that the most common and potentially harmful maternal complication of epidural anesthesia would be: A.) Severe postpartum headache. B.) Limited perception of bladder fullness. C.) Increase in respiratory rate. D.) Hypotension.

D.) Hypotension. Epidural anesthesia can lead to vasodilation and a drop in blood pressure that could interfere with adequate placental perfusion. The woman must be well hydrated before and during epidural anesthesia to prevent this problem and maintain an adequate blood pressure. Headache is not a common side effect because the spinal fluid is not normally disturbed by this anesthetic as it would be with a low spinal anesthetic. Option B is an effect of epidural anesthesia but is not the most harmful. Respiratory depression is a potentially serious complication.

Correct advice for women who ask about alcohol use during pregnancy is that it is: A.) Safest if taken during the last trimester. B.) Best to avoid it during the first 12 weeks. C.) Unknown if there is any fetal harm from its use. D.) Important to avoid it entirely throughout the pregnancy.

D.) Important to avoid it entirely throughout the pregnancy. Drinking in any amount during the pregnancy may result in adverse fetal effects. Alcohol is known to pass easily through the placental barrier, and alcohol concentrations found in the fetus are believed to be at least as high as those found in the mother. Alcohol is present in amniotic fluid, which the infant drinks.

Correct advice for women who ask about marijuana use during pregnancy is that it is: A.) Safest if used during the last trimester. B.) Best to avoid it during the first 12 weeks. C.) Unknown if there is any fetal harm from its use. D.) Important to avoid it entirely throughout the pregnancy.

D.) Important to avoid it entirely throughout the pregnancy. The active ingredient of marijuana crosses the placenta and accumulates in the fetus. Marijuana can cause hyperirritability, tremors, sleep disruption, and unusual sensitivity to light.

After an infertility assessment, a woman receives a prescription for clomiphene citrate (Clomid). The nurse should teach the woman that this medication is used to: A.) Correct excess prolactin secretion. B.) Reduce endometriosis. C.) Stimulate the release of FSH and LH. D.) Induce ovulation.

D.) Induce ovulation. Clomiphene citrate is used to induce ovulation in women who have specific types of ovulatory dysfunction. Bromocriptine (Parlodel) corrects excessive prolactin secretion. Gonadotropin-releasing hormone (GnRH) antagonists reduce endometriosis. GnRH stimulates the release of FSH and LH.

A client in active labor is to have an epidural block. While this is being administered, what nursing action takes priority? A.) Checking the uterine contractions for an increase in strength B.) Positioning the mother flat in bed to avoid post-spinal headache C.) Telling the mother she will feel the need to void more frequently D.) Monitoring the maternal blood pressure for possible hypotension

D.) Monitoring the maternal blood pressure for possible hypotension

The feature that distinguishes preeclampsia from eclampsia is the: A.) Amount of blood pressure elevation. B.) Edema of the face and fingers. C.) Presence of 4+ proteinuria. D.) Onset of one or more seizures.

D.) Onset of one or more seizures.

The nurse is planning an educational program for clients in the third trimester of pregnancy. Which childbirth education topics would be most appropriate? Select all that apply. A.) Childbirth health care provider selection B.) Morning sickness management C.) Nutritional needs during pregnancy D.) Pain relief during labor and delivery E..) Breathing techniques for labor

D.) Pain relief during labor and delivery E.) Breathing techniques for labor Rationale: - Health care provider selection is usually done in the first trimester after learning of pregnancy. - Morning sickness usually occurs in the first trimester. - Nutritional needs should have been addressed since the beginning of the pregnancy. - Childbirth education should be geared to the time in pregnancy. In the third trimester, the pregnant woman begins to focus on labor, delivery, and newborn care.

The client has come to the clinic because she suspects that she is pregnant. Which data would be most positive in confirming the diagnosis? A.) Client's report of amenorrhea for 3 months B.) Positive Hegar's sign C.) Pigmentation changes of the breasts D.) Palpation of fetal movement by the care provider

D.) Palpation of fetal movement by the care provider Rationale: Palpation of fetal movement is considered to be a positive sign because it is a completely objective sign of pregnancy that cannot have any other cause.

Some pregnant women may not be able to afford meat sources for all their meals. Which of the following food combinations will give these women a complete protein for that meal? A.) Green beans and white potatoes B.) Lettuce and tomato sandwich C.) Fresh orange slices and onions with olive oil dressing D.) Peanut butter on whole wheat bread

D.) Peanut butter on whole wheat bread Meals that contain grains and legumes provide complete proteins. Whole wheat bread (grain) and peanut butter (legume) will provide a complete protein.

The nurse reviews the client's chart for results of which diagnostic test that will best indicate a diagnosis of erythroblastosis fetalis? A.) Amniocentesis B.) Biophysical profile C.) Indirect Coombs' test D.) Percutaneous umbilical blood sampling

D.) Percutaneous umbilical blood sampling

The client's fetal heart rate (FHR) is 150 before a contraction begins. During the contraction, the FHR falls to 110 and returns to baseline 30 seconds after the contraction ends. What is the priority nursing action in response to this finding? A.) Place the client into a semi-Fowler's position. B.) Administer oxygen by nasal cannula at 2 liters per minute. C.) Insert a Foley catheter and measure urinary output. D.) Place the client in left lateral position.

D.) Place the client in left lateral position.

The client's fetal heart rate (FHR) is 150 before a contraction begins. During the contraction, the FHR falls to 110 and returns to baseline 30 seconds after the contraction ends. What is the priority nursing action in response to this finding? A.) Place the client into a semi-Fowler's position. B.) Administer oxygen by nasal cannula at 2 liters per minute. C.) Insert a Foley catheter and measure urinary output. D.) Place the client in left lateral position.

D.) Place the client in left lateral position. Rationale: With uteroplacental insufficiency repositioning the client is the best option available because it is most effective of all choices in increasing delivery of oxygen and blood flow to the fetus.

Fewer fetal movements than expected suggest possible: A.) Intrauterine fetal growth restriction. B.) Inaccurate gestational age dating. C.) Rapid intrauterine fetal maturation. D.) Reduced placental perfusion with fetal hypoxia.

D.) Reduced placental perfusion with fetal hypoxia.

The nurse makes the following assessments of a woman who is receiving intravenous magnesium sulfate: fetal heart rate (FHR), 148 to 158 bpm; pulse, 88 bpm; respirations, 9 breaths/min; blood pressure, 158/96 mm Hg. The woman is drowsy. The priority nursing action is to: A.) Increase the rate of the magnesium infusion. B.) Maintain the magnesium infusion at the current rate. C.) Slow the rate of the magnesium infusion. D.) Stop the magnesium infusion.

D.) Stop the magnesium infusion.

A healthy pregnant woman desires to increase her activity level. Which of the following is the best exercise during pregnancy? A.) Step aerobics B.) Skiing C.) Basketball D.) Swimming

D.) Swimming Swimming is an excellent form of exercise during pregnancy because the buoyancy of the water helps prevent injuries. Sports such as step aerobics, skiing, and basketball should be postponed until after birth.

When performing Leopold's maneuvers, the nurse palpates a hard round object in the uterine fundus. A smooth round surface is on the mother's right side, and irregular movable parts are felt on her left side. An irregularly shaped fetal part is felt in the suprapubic area and is easily moved upward. How should these findings be interpreted? A.) The fetal presentation is cephalic, position is ROA, and the presenting part is engaged. B.) The fetal presentation is cephalic, position is LOP, and the presenting part is not engaged. C.) The fetal presentation is breech, position is RST, and the presenting part is engaged. D.) The fetal presentation is breech, position is RSA, and the presenting part is not engaged.

D.) The fetal presentation is breech, position is RSA, and the presenting part is not engaged.

When auscultating the fetal heart rate of a term fetus during labor, the nurse notes a rate of 130 to 140 beats per minute (bpm), with occasional accelerations in the rate. How should the nurse interpret the data? A.) The baseline rate is slightly high for a term fetus. B.) Accelerations in the rate suggest intermittent hypoxia. C.) Labor usually causes the fetal heart rate to be slower. D.) These assessments are normal for a term fetus during labor.

D.) These assessments are normal for a term fetus during labor.

After administration of an epidural block for labor analgesia, the client's blood pressure decreases from 130/75 to 90/50. The nurse should assist the woman to do which of the following? A.) Lie in the supine position B.) Assume a semi-fowler's position C.) Empty her bladder D.) Turn to the side to a left lateral position

D.) Turn to the side to a left lateral position

An 8-month-pregnant client comes to the clinic complaining that she has not felt the baby move for the past 48 hours. Other than checking the fetal heart tones, the nurse can also anticipate which fetal study? A.) Amniocentesis B.) Multiple-marker study C.) Blood flow assessment D.) Ultrasonography

D.) Ultrasonography Ultrasonography is used in the last two trimesters to confirm fetal viability.

Which of the following findings meets the criteria of a reassuring FHR pattern? A.) The FHR does not change as a result of fetal activity. B.) The average baseline rate ranges between 90 and 110 bpm. C.) Mild late deceleration patterns occur with some contractions. D.) Variability averages between 6 and 25 bpm.

D.) Variability averages between 6 and 25 bpm. Variability indicates a well-oxygenated fetus with a functioning autonomic nervous system. The FHR should accelerate with fetal movement. Baseline range for the FHR is from 110 to 160 bpm. Late deceleration patterns are never reassuring.

A postpartum client has an epidural catheter in place following the delivery of an infant via cesarean section. The nurse determines that which medication is a priority to have on hand for use if needed? A.) meperidine hydrochloride B.) betamethasone C.) carboprost D.) naloxone

D.) naloxone

What specific information should the nurse include when teaching about HPV detection and treatment?

Detection of dry, wartlike growths on vulva or rectum. Need for Pap smear in the prenatal period. Treatment with laser ablation (cannot use podophyllin during pregnancy). Associated with cervical carcinoma in mother and respiratory papillomatosis in neonate. Teach about immunization for females 9 to 30 with Gardasil.

What condition should the nurse suspect if a woman of childbearing age presents to an emergency room with bilateral or unilateral abdominal pain, with or without bleeding?

Ectopic Pregnancy

What is a reactive nonstress test?

FHR acceleration of 15 bpm for 15 seconds in response to fetal movement.

What does the BPP determine?

Fetal well-being

List three necessary nursing actions prior to an ultrasound examination for the woman in the first trimester of pregnancy.

Have client fill bladder. Do not allow the client to void. Position client supine and with uterine wedge.

What are the major symptoms of preeclampsia?

Increase in BP of 30 mm Hg systolic and 15 mm Hg diastolic over previous baseline; proteinuria (albuminuria); CNS disturbances.

What instructions should the nurse give the woman with a threatened abortion?

Maintain strict bed rest for 24 to 48 hours. Avoid sexual intercourse for two weeks.

What are the major goals of nursing care related to pregnancy-induced hypertension and preeclampsia?

Maintenance of uteroplacental perfusion; prevention of seizures; prevention of complications such as HELLP syndrome, DIC, and abruption.

State three priority nursing actions in the postdelivery period for the client with preeclampsia.

Monitor for signs of blood loss. Continue to assess BP and DTRs every 4 hours. Monitor for uterine atony.

State three principles pertinent to counseling and teaching a pregnant adolescent.

Nurse must establish trust and rapport before counseling and teaching begin. Adolescents do not respond to authoritarian approach. Consider the developmental tasks of identity and social and individual intimacy.

Describe the schedule of prenatal visits for a low-risk pregnant woman.

Once every 4 weeks until 28 weeks; every 2 weeks from 28 to 36 weeks; then once a week until delivery.

What is the cause of preeclampsia?

The underlying pathophysiology appears to be generalized vasospasm with increased peripheral resistance and vascular damage. This decreased perfusion results in damage to numerous organs.

State three principles relative to the pattern of weight gain in pregnancy.

Total gain should average 25 to 35 lb. Gain should be consistent throughout pregnancy. An average of 1 lb/week should be gained in the second and third trimesters.

Identify the nursing plans and interventions for a woman hospitalized with hyperemesis gravidarum.

Weigh daily; check urine for ketone three times daily; give progressive diet; check FHR every 8 hours; monitor for electrolyte imbalances.

When should the nurse hold the dose of magnesium sulfate and call the physician?

When the client's respirations are <12/min, DTRs are absent, or urinary output is <100 mL/4hr


Conjuntos de estudio relacionados

Human Anatomy chapters 3,4,5,6,9

View Set

Chapter 16 Working with macOS and Linux

View Set

Chapter 13 - EES 1050 - Groundwater

View Set

Chapter 8: Review Questions and Exercises

View Set

chapter 5 accounting LS questions

View Set

Chapter 4: Command line interface management

View Set